support's blog

World Focus (July 2017)

World Focus (July 2017)

Magazine Name: World Focus

Month: July 2017

File Type: Printed Magazine

Order Online

Go Back to Main Page

Science Reporter (July 2017)

Science Reporter (July 2017)

Magazine Name: Science Reporter

Month: July 2017

File Type: Printed Magazine

Order Online

Go Back to Main Page

Kurukshetra Magazine (July 2017)

 Kurukshetra Magazine (July 2017)

Magazine Name: Kurukshetra Magazine

Month: July 2017

File Type: Printed Magazine

Order Online

Go Back to Main Page

CAPF-AC Solved Papers : 2016

CAPF-AC Solved Papers : 2016

Paper I : General Mental Ability and Intelligence

1. Which one of the following figures correctly represents the relations between Jupiter, Mars and Planets?

2. Consider the following. statements and conclusions.

Statements: Mohan is a good sportsman. Sportsmen are healthy.

Conclusions

I. All healthy persons are sportsmen.
II. Mohan is healthy.

Which one of the following is correct?

(a) Only Conclusion I follows
(b) Only Conclusion II follows
(C) Both Conclusions I and II follow
(d) Neither Conclusion I nor II follows

3. Which one of the following layers of atmosphere has high concentration of ions'?

(a) Stratosphere
(b) Exosphere
(c) Thermosphere
(d) Troposphere

4. Which one of the following cities has greater annual range of temperature?

(a) Kuala Lumpur
(b) New Delhi
(c) Shanghai
(d) Ulaanbaatar

5. Match list I with list II and select the correct answer using the code given below the lists.

List I                                 List II
(Climate type in             (Major area)
letter code)

A. Aw                                 1. Central California
B. Af                                   2. India Sub-continent
C. Cs                                  3. North and South of Amazon Forests
D. Am                                4. Western Equatorial Africa
 

Code

    A B C D
(a) 3 1 4 2
(b) 2 4 1 3
(C) 3 4 1 2
(d) 2 1 4 3

6. Ramsar Convention, to which India is a signatory, is related to conservation and wise use of

(a) wetlands
(b) genetic resources
(c) mines
(d) forest lands

7. Which one of the following rivers flows between Satpura and Vindhya ranges?

(a) Tapi
(b) Sabarmati
(c) Narmada
(d) Mahi

8. Which one of the following is the correct taxonomic hierarchy?

(a) Kingdom-Phylum-Order-Genus-Family-Class-Species
(b) Kingdom-Order-Class-Phylum-Family-Genus-Species
(c) Kingdom-Class-Order-Phylum-Family-Species-Genus
(d) Kingdom-Phylum-Class-Order-Family-Genus-Species

9. The subunits of DNA are known as

(a) Nucleotide
(b) Nucleosome
(c) Nucleoside
(d) Polypeptide

10. The process of using microbes to treat areas of land or sea that have been contaminated by pesticides, oil or solvents is known is

(a) Eutrophication
(b) Nitrification
(c) Ammonification
(d) Bioremediation

11. How is the rate transpiration affected by decreasing humidty and by decreasing light intensity?

Decreasing                 Decreasing
humidity                     light intensity

(a) Decreases             Decreases
(b) Increases              Decreases
(c) Increases              Increases
(d) Decreases            Increases

12. Vaccination involves

(a) Injecting the body with materials that simulate the body to produce antibodies
(b) injecting the body with materials that stimulate the body to produce antigens
(c) the use of monolonal antibodies to cure a disease
(d) use of antibotics to cure diseases

13. Which of the following are examples of carnivorous plants?

(a) Sundew, Venus fly trap, Pitcher plant
(b) Cuscuta, Raffesisa, Mistletoe
(c) Sandalwood, tree Broom rape, Pitcher plant
(d) Cuscuta, Bladderwort, Mistletoe

14. Consider the foollowing.

15. Consider the following diagram (not in scale).

There are seven places marked as P, Q, R, S, T, U and V as shown in the diagram. The directly connected paths between two places are indicated by line segments joining thet wo places alongwith the length labelled (in km). Then, the shortest distance between P and U is

(a) 14
(b) 15
(c) 12
(d) 13

16. The number of persons reading newspaper is shown in the following Venn diagram (Survey of 50 persos)s.

In a population of 10000, what is the number of persons expected to read atleast two newspapers?

(a) 5000
(b) 6000
(c) 6250
(d) 5400

17. A vehicle with mileage 15 km/L contains 2 L of fuel. The vehicle gets some defect asa result of which 5 L of fuel gets wasted per hour, when the engine is on. With what minimum speed, the vechile has to move to travel 20 km with the existing amount of fuel, if it travels with a uniform speed?

(a) 100 km/h
(b) 120 km/h
(c) 150 km/h
(d) 200 km/h

18. Consider the following statemetns relating to Sea Salinity.

1. The ocean salinity depends on evaporation and precipitation.
2. Any change in the temperature or density influences the salinity.
3. Major source of sea salinity of terrestrial discharge by rivers.

Which of the statements given above are correct?

(a) 1 and 2
(b) 2 and 3
(c) 1 and 3
(d) 1, 2 and 3

19. Consider the following statements relating to salt production in India.

1. India is the second largest producer of iodised salt in the world next only to China.
2. Salt mining is carried out in Himachal Pradesh.
3. Gujarat is the leading producer of salf in India.
4. Ground water is the important source of salt in Rajasthan.

Which of the statemetns given above are correct?

(a) 1 and 2
(b) 3 and 4
(c) 1, 2 and 3
(d) 2, 3 and 4

20. Which of the following statements relating to Dharwar geological system are correct?

1. They belong to Archaean geological period.
2. They re metamorphosed sedimentary rocks.
3. They are of great economic importance for its mineral resources.
4. They are found predominintly in Karnataka, Tamil Nadu, Odisha and Jharkhand.

Select the correct answer using the code given below.

(a) 1, 2, 3 and 4
(b) 2, 3 and 4
(c) 3 and 4
(d) 1 and 2

केन्द्रीय सशस्त्र पुलिस बल (सहायक कमांडेंट) के लिये स्टडी किट

Study Kit for Central Armed Police Forces

Current-Public-Administration-Magazine-(June-2017)-Speaking for the voiceless


Sample Material of Current Public Administration Magazine

Current Development


Speaking for the voiceless

                                        - R.K. Vij

Recently an eight-year-old girl was reportedly raped by the father of her 13-year-old friend in broad day light in Delhi. This act was apparently witnessed by the victim’s friend from a distance. Both were so deeply traumatised that they are undergoing counselling. Such cases are not uncommon in India. The fate of such cases can be predicted: NCRB data shows that almost three-fourths of rape cases go unpunished every year.
The witness, in all probability, may not speak against her father. Quite often, doctors, who conduct the primary examination of the victim, depose in the court that the injury caused to the minor could be by any other object. To what extent an eight-year-old would be able to explain the act of sexual assault in court is often the only deciding factor to prove the rape charges.

An analysis of rape cases of minors reveals that nothing has changed even after the enactment of the Protection Of Children from Sexual Offences (POSCO) Act in 2012. Section 29 of this law says that “the special court shall presume that the person prosecuted under sections of penetrative sexual assault has committed or attempted to commit the offence unless the contrary is proved”. However, experience reveals that the prosecution is still asked to prove the case “beyond reasonable doubt”.

The law permits the medical examination of minor victims only with guardians’ consent. If such consent is not granted, more emphasis needs to be laid on oral evidence. However, it has been found that comparatively less importance is given to the uncorroborated oral evidence in cases where the consent for medical examination is withheld. The scrutiny of evidence in such cases is generally the same as in the cases where the victim is fully aware of the criminal act. Such standards need to be considered afresh.

Criminal justice emerges out of the friction between two adversaries — the prosecution and defence — with the judicial magistrate acting like an impartial referee. However, in cases involving minors, judges may need to be slightly pro-active, as even the SC has criticised the passive role played by lower court judges.

News Bulletin 5: Minor Girl Raped & Murdered; Minor Girl

The Justice Malimath Committee, constituted to suggest measures to reform the Indian criminal justice system, stated in its report that the adversarial system of dispensing justice had not worked satisfactorily in India and some beneficial features of the inquisitorial system should be incorporated. A few high courts and state governments had also agreed with the committee’s suggestion. In an inquisitorial system, (as applicable in China, Russia, Japan, Scotland etc) judicial magistrates investigate criminal offences and search for the truth. In Germany, a breach of the judges’ duty to actively discover truth would amount to a procedural error which may provide grounds for an appeal. Italy uses a blend of two systems.

Some elements of the inquisitorial system are already incorporated in the Indian law. Section 176 of the CrPC says that custodial death or disappearance and custodial rape shall be inquired into by the judicial magistrate in addition to the inquiry or investigation held by the police. Investigative powers can be given to the judicial magistrates in cases of rape of children under 12 years of age. The investigative judicial (woman) magistrate could be assisted by a female medical doctor and an expert in child psychology. The report of such magistrate could be treated as a final piece of evidence and the accused may be given an opportunity to state and defend his version.

Having a separate law like the POCSO Act does not seem to be sufficient to dispense justice to a spectrum of victims who are not even capable of comprehending the trauma of sexual violence. Radical changes are required to deal with such offences. It might be the time to introduce the inquisitorial system in cases of sexual exploitation of children who cannot speak for themselves.
(Source- The Indian Express)

Get this magazine (Current Public Administration) free if you purchase our any of the below courses:

Study Kits For Public Administration

Online Coaching For Public Administration

<< Go Back to Main Page

Current Public Administration Magazine (June - 2017) - Make transparency a patriotic duty


Sample Material of Current Public Administration Magazine

Ethics and Integrity


Make transparency a patriotic duty

                                        - Venkatesh Nayak

Soon after the GST regime was rolled out, the Hon’ble Prime Minister announced that the Registrars of Companies (RoC) had struck 100,000 shell companies off their rolls. Addressing chartered accountants in Delhi on July 1, 2017, Mr Narendra Modi said this historical decision was a sterling example of how tough decisions can be taken if “patriotism” served as the inspiration.
The 3,000-strong gathering welcomed this announcement with thunderous applause and the media reported it as another step in the direction of curbing the influence of black money, stemming the tide of corruption and cleaning up the economy, as part of the demonetisation exercise that was launched on November 8, 2016.

So far, so good. This author spent more than an hour looking for the list of companies struck off the rolls by Delhi’s RoC, in vain. The website of the Ministry of Corporate Affairs only displays a public notice from April this year, inviting objections from any person on the proposal to terminate more than 26,000 registered companies for being “defunct” and not seeking ‘dormant’ status formally under the law.
This is not to doubt the veracity of the Hon’ble PM‘s announcement. Instead this is one of the indicators of how compliant public authorities are with the transparency requirements of the Right to Information Act (RTI Act). This 12-year old law places a duty on every public authority to place all relevant facts and figures while announcing decisions that affect the public at large. The ‘termination’ of shell companies eminently qualifies to such treatment. Yet, there is very little detailing of action taken by the RoCs.

Since October, 2005 almost every government has breached its obligation of transparency repeatedly. The bureaucracy has fought shy of graduating from transparency on a “need to know basis” (where the government decides what people should or should not know) to openness founded on the “people’s right to know” as streamlined by the RTI Act.

It comes as no surprise that the government has repeatedly refused to disclose details about the November 2016 demonetisation exercise as well, despite attracting enormous public support. The citizenry is required to applaud and cheer every decision but it simply cannot be trusted with the details of the decision making process, it seems.

Making sense of the “method” behind the treatment meted out towards transparency by the erstwhile UPA and the current NDA regimes is difficult to rationalise. Although the enactment of the RTI law was hailed by many as a historic juncture, next in importance only to India’s independence from colonial rule, and the UPA Government showcased it as one of its biggest achievements, the then Prime Minister certainly did not seem convinced of its value. On several occasions, Dr. Manmohan Singh voiced his concerns about the “misuse” of the RTI law although it turned out later that his office did not have any factual data to back up this claim.
Even before the RTI law could reach its first anniversary, the UPA Cabinet decided to amend it to keep file notings or the deliberations of bureaucrats and ministers in any decision-making process away from the public gaze. Thanks to the very vocal opposition from those who had used the RTI law to dramatic effect to expose corruption and wrongdoing at various levels, this proposal had to be shelved.

Nevertheless, the UPA was instrumental in setting up a gigantic data portal that enabled any person to track the manner of implementation of the Mahatma GandhiRural Employment Guarantee Act in real time. Few initiatives taken by governments in other developing countries could match this transparency initiative. This was not all. Tour reports of ministers and senior bureaucrats visiting abroad were required to be made public, MOUs with and earnings of public private partnerships were made transparent and templates were issued to make panchayats, government schools and hospitals in villages to volunteer information about their working.
In complete contrast with his predecessor, Prime Minister Modi is the most vocal advocate of transparency that India has seen till date. Not only did the NDA make transparent and accountable government as its springboard to capture power, while celebrating the 10th anniversary of the RTI Act, the PM urged the public authorities to study the trend of information requests from citizens. Bureaucrats were advised to examine what categories of information people were seeking again and again and if they could be disclosed voluntarily so that people’s need to seek information through RTI applications is progressively reduced- in fact the very purpose of the RTI Act. Till date, no such exercise has been reported by any ministry or department at the Central level. The bureaucracy seems to be sleeping over this commonsensical and practical idea.

Next, in April, 2016, the Central Information Commission (CIC) issued a progressive recommendation requiring the Government to publicise the details of work done by every ministry and department. The Cabinet Secretariat took uncharacteristically rapid action on this recommendation and issued a circular to all ministries requiring them to publish reports of major achievements, events and occurrences and decisions taken every month. More than a year has passed since then, compliance remains extremely poor (less than 10% of the ministries are complying with this directive). In fact, the Government confessed that it did not have a mechanism to monitor the implementation of its own circular, when quizzed by two members of the Lok Sabha in March, this year.

Or take the case of the Naga framework agreement signed in August 2015 with much fanfare. Reporting on this big breakthrough where at least one Naga faction agreed to come to the discussion table, the Government assured that details of the agreement would be publicly disclosed soon. Almost two years have passed, but the Government has stubbornly refused to disclose any details of this agreement despite the topic coming up for debate again and again, most recently, during the recent Assembly elections held in Manipur.

The CIC also upheld the decision to keep the entire matter under wraps in a case filed by the author. While some Naga spokespersons are openly commenting on the content of the negotiations, the citizen-taxpayer, it seems, has to wait for the official will to turn in favour of transparency to ascertain the truth behind their claims.

It is not as if the NDA Government has paid only lip sympathy for transparency by preaching from the pulpit without any follow-up. Soon after coming to power the NDA Government amended the conduct rules applicable to bureaucrats of all ranks- making transparency and accountability as core values that should inform their working. Nevertheless, the policy level commitment to transparency has not translated into actual practice at the cutting edge level of the administration belying the promise of open and minimum government.

Why is the bureaucracy fighting back against transparency instead of toeing the line? Why is the political executive unable to get the administration to deliver on this commitment? This contrast between the words and the actions of the UPA and NDA Governments requires in-depth study.

In the interim, it might help if transparency were declared a “patriotic duty” to secure compliance from the administration. Few other values seem to be attracting as much attention and action in these turbulent times.

(Source- The Indian Express)

Get this magazine (Current Public Administration) free if you purchase our any of the below courses:

Study Kits For Public Administration

Online Coaching For Public Administration

<< Go Back to Main Page

Public Administration Global Journals : Origin and theoretical basis of New Public Management

Public Administration Global Journals : Origin and theoretical basis of New Public Management

The NPM movement began in the late 1970s and early 1980s. Its first practitioners emerged in the United Kingdom under Prime Minister Margaret Thatcher and in the municipal governments in the U.S. (e.g., Sunnyvale, California) that had suffered most heavily from economic recession and tax revolts. Next, the governments of New Zealand and Australia joined the movement. Their successes put NPM administrative reforms on the agendas of most OECD countries and other nations as well (OECD, 1995).

The development of administrative thought in the United States

The conscious study of public administration in the U.S. began in a time when its public administration was in a state of disrepute. In the late 19th century, the administrative mechanisms in the U.S. were dominated by the spoils system—administrative positions were distributed to those who contributed to the victorious party’s electoral success. Administra- tive personnel, therefore, changed frequently. Incompetence, inefficiency, and corruption were common (Weber, 1956, p. 839ff; Van Riper, 1987; Stone and Stone, 1975; Schachter,
1989).

In reaction to this corruption, the Progressives created a movement to reform politics and administration, pressing for a more interventionist state, the separation of politics and administration, the merit principle (tenured, neutral, and competent administrators), and sound financial management. The Progressives achieved several notable successes: a career civil service (Pendleton Act, 1883), line-item budgets, and less political partisanship and corruption (Eisenach, 1994; Lee, 1995; Waldo, 1948).

Classical public administration

In the Progressive movement, the New York Bureau for Municipal Research was a key player. Influenced by Frederick Taylor’s scientific management, the New York Bureau believed that efficiency was the best solution to the problem of corruption and incompetence. These progressive reformers imported techniques and studies from scientific management (e.g., on efficient street paving and snow removal). They were the first to use performance indicators to benchmark the efficiency of public organizations, one purpose of which was to identify corruption (Schachter, 1989). In the 1920s, some practitioners and academics created the science of public administration on the fundamentals of the progressive reform successes—particularly the presupposition of loyal bureaucrats, honest politicians, and the politics-administration dichotomy. These reformers—the new scientists of public adminis- tration— built a theory of organization that they supplemented with the concept of manage- ment.
These principles were:

Y The principle of division of work and specialization.
Y The principle of homogeneity.
Y The principle of unity of command.
Y The principle of hierarchy with respect to the delegation of authority.
Y The principle of accountability.
Y The principle of span of control.
Y The staff principle (Gulick, 1937; Urwick, 1937; Mooney, 1937; Graicunas, 1937).

The reformers expected public managers, working within organizational structures built on these principles, to perform the following functions: Planning, Organizing, Staffing, Directing, Coordinating, Reporting and Budgeting— or, in Luther Gulick’s shorthand: POS- DCORB (Gulick, 1937, p. 13). The reformers also advocated reorganization to streamline and consolidate organizations and to standardize administrative procedures (Lee, 1995; Henry, 1975; Arnold, 1995).

During the New Deal of the 1930s, the scope of government activity and the public administration of the U.S. was dramatically expanded— but still guided by the principles of public administration. The New Deal followed (and realized) the societal vision of the progressives. Government in the U.S. became more involved: it regulated more activities; it followed social-democratic ideals; it seemed to be built on scientific objectivity; and it promised material freedom (Egger, 1975; Waldo, 1948; Van Riper, 1987).

I call this cluster of organization and management concepts and the closely related ethos of orderly government an active state, and label the belief in objective knowledge that serves to control the social and physical environment classical public administration.

Neoclassical public administration

After World War II, academics began to reassess and question the principles of classical public administration. One of the most rigorous critics was Herbert Simon, whose work set the tone and direction for neoclassic public administration. His dissertation, with the title Administrative Behavior: A Study of Decision-Making in Administrative Organization, contained the buzzwords of the era: behavior, decisions, and organization.
Simon said that the principles of administration are not scientific, but inconsistent prov- erbs that were drawn from common sense (Simon, 1976). He suggested founding public administration on rigorous and scientific observation and on (inductively) derived laws of human behavior. He advocated separating facts from value judgments and dividing science into pure and applied branches (Simon, 1976; Simon, Smithburg, & Thompson, 1962). From this perspective, objective scientific knowledge serves to control the social environment.

Simon’s ideas significantly influenced a host of scholars who undertook studies of behavior and decisions in administrative organizations and created a new, more precise vocabulary and research methodology. This neoclassical public administration (and Simon himself) followed the common trends of behaviorism, structural functionalism, and systems theory, and employed the theoretical underpinnings of welfare economics and decision theory.

Nevertheless, during this time, governmental reformers continued to follow the Progres- sives’ ideals and classical theory: organizations and jurisdictions were streamlined and consolidated; executive power was strengthened and unified.

Still, the main practical event of this period was the invention of the Program, Planning, and Budgeting System, or PPBS (Waldo, 1969). PPBS was based on microeconomic decision techniques and a strong belief that central planning of the national administration (and economy) could lead to successful optimization. PPBS applied the logic of systems analysis and rational planning, and employed the systems-theoretical vocabulary of inputs, throughputs, outputs, outcomes, programs, and alternatives (Schick, 1966; Greenhouse, 1966; Gross, 1969; Schick, 1969). Unfortunately, the implementation of the PPBS suffered serious shortcomings, the system never worked as intended, and in 1972, the U.S. govern- ment formally terminated it (Schick, 1973). So, while neoclassical public administration seemed to alter its standards and methodology, practice continued to rely on the principles of the classics and on the administrative strictures of the Progressives (Lynn, 1996; Kramer, 1987). Thus, advocates of the neoclas- sical approach adapted themselves to the principles and structures of classical public administration and carried them on, improving the analytical basis for performance mea- surement, auditing, budgeting, and the rationalization of jurisdictions and organizations. What they especially added was a focus on analysis and a shift from a bureaucratic management style toward a more rational and analytic one. But there were other indirect consequences of the neoclassical approach. Social science research about organizations, behavior, and leadership was finding that human beings are complex (Schein, 1965), that modes of leadership other than the classical, directive style are possible, and that organizations can be structured not only in mechanistic but also in organic ways (Burns and Stalker, 1971). These findings didn’t have direct, practical, consequences, but they were generating interest in the academy.

Another consequence of neoclassical reassessment was that public administration lost its unity (Waldo, 1965). A lot of scholars did follow Herbert Simon’s lead— but not all. Some felt unqualified to use the new scientific standards and thus continued doing what they did before; so the classical approach not only survived in the Progressive structures of practical government but in public-administration theory, too. Other scholars refused to accept the separation of facts and values, because they thought this would cut off public administration from its foundations—from political philosophy and the search for the public interest (Waldo, 1965; Subramaniam, 1963). Thus, at the end of the 1960s, the field of public administration included a classical line of thought, a neoclassical line of thought, and a group of politically oriented scholars. Still, these divided groups did share one common creed: the Progressive vision of an active state and belief in objective knowledge. But other scholars were creating new approaches that seriously questioned this basic belief of public-administration scholars.

Public choice and modern institutional economics

The first of these rival approaches was public-choice theory. The main event of the institutionalization of the public-choice approach was the founding of the Thomas Jefferson Center for Studies in Political Economy and Social Philosophy by James Buchanan and Warren Nutter at the University of Virginia. Buchanan and Nutter wanted to build a platform for all scholars who were interested in a society based on individual freedom (Buchanan, 1986). Furthermore, Buchanan, Nutter, and their colleagues employed methodological indi- vidualism as their basic theoretical approach. They sought to explain social phenomena by aggregating the behavior of individuals. This approach is based on the assumption that individuals pursue their own aims and act according to their preferences. It assumes a different concept of rationality than Simon did. From the public-choice perspective, rationality is not bounded compared with a theoretical optimum; rather, rational behavior is when a person acts to pursue his or her aims according to his or her knowledge of the situation. For example, a Native-American who believes that raindancing produces rain and who begins to dance in a severe drought is behaving rationally (Tullock, 1965). Public-choice theorists deductively develop models to explain social phe nomena from a set of assumptions about individuals’ aims and their information about their situations.

Fig. 1. The logic of individualistic explanation (Coleman, 1990).

This approach also created a natural normative benchmark for assessing social reality. Because individual preferences and the free choice of individuals are central to the public- choice approach, its benchmark for political institutions is whether a free individual would willingly agree to such structures and to their outcomes (Buchanan and Tullock, 1962; Buchanan, 1975). One could call this a contractarian vision of the state and its constitution.

Using this framework, public-choice scholars assessed the modern welfare state that the Progressives and their public-administration companions built. Their conclusions were highly critical. Their theoretical explanation of representative democracy found that the use of the simple majority rule without constitutional safeguards can lead (through logrolling) to the exploitation of minorities by majorities, that such majorities have an incentive to waste resources that the minority pays for, and that the notion of a public interest or a common good postulated by classical democratic theory is highly questionable (Downs, 1957; Buchanan and Tullock, 1962). Public-choice scholars further showed that tendencies for inefficient use of resources and the exploitation of certain groups are enhanced by traditional budgeting by representative committees, and by the executives within bureaucratic organi- zations (Niskanen, 1971). They also demonstrated that bureaucratic organizations (defined as organizations that are partly or wholly not evaluated on markets) have a number of serious deficiencies. From a public-choice perspective, these deficiencies include a strong tendency toward the accumulation of tasks and resources, toward excessive conservatism, and toward a law-like inability to accomplish certain tasks (Downs, 1966; Tullock, 1965).

According to their explanation of state failure and based on their individual ethical premises, public-choice scholars made a number of reform proposals. Their main issues were  the invention of constitutional safeguards against exploitation (political exploitation can be assumed if the sum of taxes an individual pays is greater than the value of public goods he or she receives) and the invention of a polycentric administrative system (in contrast to the monocentric system of the Progressives). In such a polycentric administrative system, the provision and the production of services are separated. Private vendors and public vendors compete1 for production contracts.

The size of production units and public consumption units (jurisdictions) are not neces- sarily identical (allowing economies of scale and efficient intergovernmental contracting, and contracting between public consumption units and production units, which belong to other public consumption units). Such a system would function best if it was highly decentralized and federal. Thus, a polycentric system would use transparent financing systems such as user charges, vouchers for public goods, and opportunities for citizens (as consumers) to make choices (Ostrom, 1973, 1977; Savas, 1982).
In a polycentric system a lot of individuals would make decisions according to their personal preferences and knowledge; there would not be a single (and usually distant) decision center, but many of them (Hayek, 1960, 1969). Every component of public-choice theory—the methodology, the ethical benchmarks, and the recommendations— directly con- flicts with classical and neoclassical public administration. Vincent and Elinor Ostrom presented this as a new approach to public administration and found some supporters (Ostrom and Ostrom, 1971).

Other approaches of modern institutional economics include neo-Austrian economics, property-rights theory, principal-agent theory, and transaction-costs economics. Neo-Aus- trian economists deal mainly with the dilemmas of planned and unplanned social order and share much of the public-choice perspective (Hayek, 1960, 1969). Neo-Austrian economists have a very strong preference for individual freedom, and stress the necessity of laws (including tax laws) that do not discriminate in any way. They oppose the welfare state, which they view as a form of tyranny (Mises, 1944). Property-rights theory deals with the efficient allocation of property rights over resources (Demsetz, 1967). Principal-agent theory focuses on the problems that superiors have in monitoring the behavior of their subordinates and in creating incentives for subordinate behavior (Pratt and Zeckhauser, 1985). Transac- tion-cost economics mainly deal with the question of when markets or hierarchies are used as efficient arrangements for the organization of production (Williamson, 1975).
All of these approaches present a major challenge to both classic and neoclassic thinking.

Public Administration Study Kit For Mains Examination

Public Administration Mains Test Series 2017-18

Current Public Administration Magazine (June - 2017) - A Litmus Test


Sample Material of Current Public Administration Magazine

POLITY AND GOVERNANCE ISSUES


A Litmus Test

                                        - Faizan Mustafa

In India, people have the right to life, but fake encounters and mob lynching happen. In spite of the right to free speech, publications feel compelled to withdraw articles critical of government or corporates. There is a right to equality but discrimination is still rampant.

When the mention of fundamental rights in the Constitution is not able to ensure their full implementation on ground, one wonders what will happen if privacy is not recognised as a fundamental right. In such a situation, citizens may not have protection against surveillance and even profiling by the state, the state could target those who speak against it, even voting preferences may be influenced, telephone tapping could be routinely resorted to and our mails intercepted. This is indeed a terrifying prospect.

The right to privacy is not explicitly mentioned in the Constitution. But then the right to “due process” too was not there and, in fact, was dropped by the framers of the Constitution. Yet, the apex court read it into the “right to personal liberty”. The court, in fact, silently brought about what may be called a “rights revolution” by judicially creating several fundamental rights.
When the democratic state turned totalitarian under Indira Gandhi and started abusing its powers to amend the Constitution, the Supreme Court as protector of civil liberties stood firm and applied the brakes first, in 1967, by denying Parliament power to amend the Constitution and then, in 1973, through the “basic structure” doctrine which too is not there in the text of the Constitution.
If the text of the Constitution alone is going to determine the nature of the right to privacy, then the collegium system, the right against arbitrariness and the freedom of press too could go soon. Voluntary surrender of personal information to private entities cannot be equated with mandatory data collection by the state. The right to privacy judgment will be a litmus test for the apex court.
Will the court follow the rich traditions of 1967 and 1973 and rise to the challenges of the information age? One hopes there would not be another ADM Jabalpur (1976) kind of decision where the majority accepted the government’s argument that when the right to life and personal liberty is suspended, citizens have no remedy against illegal detention.

It is erroneous to believe that eight- and six-judge benches have authoritatively held that there is no right to privacy. In the Satish Chandra case (1954), the fundamental question was whether the state’s power of search and seizure violated the right against self-incrimination under Article 20(3). In a positivist mould, the court refused to read right to privacy under this provision.

Then came the Kharak Singh (1963) case where a dacoity accused was released and put under surveillance. Police constables would knock at his door, wake him up during night and disturb his sleep. The majority conceded that “everyman’s home is his castle” and struck down domiciliary visit regulations. But without any elaborate discussions, the court yet again said that there was no fundamental right to privacy in India.
But there was the powerful dissenting judgment of Justice Subba Roa, with whom Justice J.C. Shah concurred. They argued that even though the right to privacy is not specifically mentioned in the Constitution, it is a necessary ingredient of the right to personal liberty. In the Gobind case (1975) the minority opinion of Kharak Singh case became the majority opinion. The court has recognised right to privacy as an integral part of right to personal liberty. Today, liberty is a part of the basic structure of the Constitution.

Despite the recognition of privacy as a fundamental right, the government will continue to have powers to impose “reasonable restrictions”. It is no body’s case that the right to privacy is an absolute right. Moreover, global experience shows that the denial of privacy neither promotes national security nor curbs terrorism, it merely takes away citizen’s freedom to be left alone and curtails his/her choice in personal decisions.

(Source- The Indian Express)

Get this magazine (Current Public Administration) free if you purchase our any of the below courses:

Study Kits For Public Administration

Online Coaching For Public Administration

<< Go Back to Main Page

Current Public Administration Magazine (June - 2017) - States of distress


Sample Material of Current Public Administration Magazine

Centre State Relations


States of distress

                                        - The Indian Express Editorial

The combined fiscal position of states has deteriorated sharply over the last couple of years, with the gross fiscal deficit-GDP ratio breaching the 3 per cent threshold in 2015-16 — the first time this has happened in over a decade. While this marks a reversal of the trend till two years back, when states were more fiscally prudent than the central government, it is worrying that the run-rate is likely to continue and possibly worsen — triggered by a combination of revenue and expenditure side issues.

The spate of farm loan waivers and the expected rise in the interest liabilities of states that have participated in financial restructuring of electricity distribution utilities (through the UDAY scheme), are two looming stress points. On the receipts side, while grants from the Centre have gone up — reflecting the changes in the pattern of funding of centrally sponsored schemes — there is likely to be fresh strain on the receipts side as nearly a dozen states are projected to see a sharp dip in state excise revenues from alcohol due to the closure of bars along highways and efforts at prohibition.
States’ revenues from stamp duties, which are about 12 per cent of overall state revenues, have also seen a slowdown in the backdrop of the continuing impact of demonetisation on property sales. The RBI has strongly highlighted these risks, and two additional stress points — the guarantee commitments of state governments with respect of state public sector enterprises and the potential increase in states’ committed liabilities in case they decide to implement the recommendations of their own Pay Commissions in FY’18.

Hyderabad: State Budget Will Strengthen Rural Economy

As a result, even as the Centre’s fiscal deficit has progressively come down to 3.5 per cent in FY’17 from 4.9 per cent in FY’13, the combined fiscal deficit (of Centre and states) could touch 7 per cent this fiscal due to bloating state deficits. The Centre has to face its share of the blame, especially on the farm loan waivers that got traction in the run-up to the UP elections, setting off cascading demands across other states. The UP government’s announcement of a Rs 36,000-crore farm loan waiver scheme came despite the fact that it figures in a list of states that are not eligible for additional market borrowing as they are not compliant with fiscal prudence norms prescribed by the fourteenth finance commission.

Maharashtra, which had announced a Rs 34,000-crore crop loan waiver for farmers last month, is staring at the prospect of the state’s public debt topping the Rs 4 lakh-crore mark by March 2018. So, even as the Union government has budgeted for a sharply lower fiscal deficit at 3.2 per cent this financial year, the continued fiscal profligacy of states would be counterproductive to the Centre’s consolidation exercise in the longer run.

(The Indian Express Editorial)

Get this magazine (Current Public Administration) free if you purchase our any of the below courses:

Study Kits For Public Administration

Online Coaching For Public Administration

<< Go Back to Main Page

Current Public Administration Magazine (June - 2017) - Cleaning the House


Sample Material of Current Public Administration Magazine

Accountability and Control


Cleaning the House

                                        - S Y Quraishi

Criminalisation of politics has been a matter of great concern, particularly in the last two decades. The Supreme Court’s latest order asks the Election Commission to clarify its position on the PIL seeking a lifelong ban on candidates convicted in criminal cases. Just a couple of months ago, the EC had supported the petitioner. In the hearing on July 18, however, the EC’s advocate took a different stand.
“[The] Commission has not taken any decision and doubts whether it falls in the legislature’s domain,” he told the court. Since this stand was inconsistent with the one taken earlier, the Supreme Court came down heavily on the EC. “Can you afford to remain silent when it is within the domain of the commission? If you don’t want to be independent, if you want to be constrained by the legislature, constrained even to express your views, say so freely,” it said.

Such a rebuke to the commission is unprecedented. The Supreme Court, in fact, has been extremely supportive of the EC, so much so that I have always described it as the commission’s guardian angel. When the matter comes up before the court next, it is expected that the EC will unequivocally reiterate the plea to debar convicted MPs and MLAs for life from contesting elections. “The Commission has not taken any U-turn and is unambiguously in sync with the petitioner’s prayer to debar the convicted legislators (MPs and MLAs) for life,” the commission’s officials have clarified.
My personal opinion is that the lifelong ban is too harsh. Under the present law, the minimum bar of a politician from election is eight years (two years of minimum imprisonment followed by six years of ban). That is enough to kill his political career.

The positive aspect of the unpleasant situation is that the need for cleansing politics of criminal influence has been flagged once again. The issue has two dimensions: One, how to deal with the politicians after conviction and, two, before conviction, while they are still under trial.

Protecting the parliamentary system from criminalisation has been the intention of the law from the beginning. Section 8 of the Representation of the People (RP) Act, 1951 disqualifies a person convicted with a sentence of two years or more from contesting elections. But those under trial continued to be eligible to contest elections. The Lily Thomas case (2013), however, ended this unfair advantage.
The SC has repeatedly expressed concern about the purity of legislatures. In 2002, it made it obligatory for all candidates to file an affidavit before the returning officer, disclosing criminal cases pending against them. Civil society organisations like the Association for Democratic Reforms (ADR) analyse and publish this information. The concern of the apex court has been increasingly clear from a series of judgments. The famous order to introduce NOTA was intended to make political parties think before giving tickets to the tainted. According to the ADR’s analysis of EC data, 187 MPs in the current Lok Sabha face criminal charges (that is, 34.4 per cent). Of them, 113 face serious criminal charges. The number has gone up from 162 (76 serious) charges in 2009 and 128 (58 serious) in 2004. This obviously shows the political class and legislature in a poor light. What is the way out? There are three possible options. One, political parties should themselves refuse tickets to the tainted. Two, the RP Act should be amended to debar persons against whom cases of a heinous nature are pending from contesting elections. Three, fast-track courts should decide the cases of tainted legislators quickly.

Far from denying tickets to criminals, all political parties seem to have been in competition to give them more and more tickets. They have been unitedly opposing the proposal to debar perpetrators of even heinous offences during pendency of trial on the grounds that false criminal cases may be filed by opponents. This concern is partly valid. However, the EC’s proposal itself has three safeguards. First, all criminal cases would not invite the ban — only heinous offences like murder, dacoity, rape, kidnapping or moral turpitude. Second, the case should have been registered at least a year before the elections. Third, the court must have framed the charges.

The opponents of the proposal also argue that the jurisprudence followed in the country holds that a person is innocent until proven guilty. Then what about the over 2.7 lakh persons today who are undertrials (and are therefore “innocent”) and locked up in jails? Their fundamental rights of liberty, freedom of movement, freedom of occupation and dignity are denied. Does this not violate the fundamental right of equality? If the rights of an undertrial can be suspended within the ambit of the law, what is the problem about temporarily suspending the right to contest an election, which, incidentally, is only a statutory right?
Another pertinent question is will fast-track courts be illegal as they will treat the cases against politicians differently from the lakhs of other cases awaiting a decision for years? No. Fast tracking has been the accepted norm. Special CBI courts, consumer courts, special courts for economic offences and, more recently, fast-track courts for rape cases do create special categories for adjudication, and nobody has dubbed them discriminatory.

The RP Act also recognises the principle of fast-tracking, requiring the high court to decide on election petitions within six months. The fact that courts have been taking years to decide on election petitions makes it an issue of dereliction by the judiciary of its statutory obligation, not a deficiency of the law.
This has been addressed by the landmark judgment of March 2014, in which the SC accepted the urgent need for cleansing politics of criminalisation and directed all subordinate courts to decide on cases involving legislators within a year, or give reasons for not doing so to the chief justice of the high court. The government of India promptly offered full support for the implementation of this order. It is important to see how the SC judgement has been implemented on the ground.

With the SC positively inclined and the PM’s concern for a corruption-free India through electoral reforms, the problem may finally be nearing resolution. The writer is former chief election commissioner of India, and author of ‘An Undocumented Wonder — The Making of the Great Indian Election’

(Source- The Indian Express)

Get this magazine (Current Public Administration) free if you purchase our any of the below courses:

Study Kits For Public Administration

Online Coaching For Public Administration

<< Go Back to Main Page

CDS Exam Solved Papers - 2012- I Paper II : General English

CDS Exam Solved Papers - 2012- I Paper II : General English

Paper II : General English

SELECTING WORDS

Directions (Qs. 1-12): In the following passage, at certain points you are given a choice of three words in a bracket, one of which first the meaning of the passage. Choose the best word from each bracket. Mark the letter, viz A, B or C, relating to this word on your Answer Sheet. Examples Y and Z have been solved for you.

Y. the A. boy was in the school in Shimla
B. horse
C. dog

Z. A. She was homesick
B. it
C. He

Explanation: Out of the list given in item Y, only boy is the correct answer because usually a boy, and not a horse or a dog, attends school. So a is to be marked on the Answer Sheet for item Y. A boy is usually referred to as ‘’he’’, so for item Z, the letter C is the correct answer. Notice that to solve the first item Y you have to read the rest of the sentence and then see what fits best.

PASSAGE A

1. According to a report in yesterday’s newspaper

A. Once police dog was taken to Raj Bhava
B. A
C. New

2. A. at Monday. This was to trace the
B. Next
C. On

3. A. killers, of the “very important horse” which
B. Dogs
C. Police

4. A. has
B. were
C. was

5. Reported missing on Sunday. The dog picked

A. on the scent on some traces of
B. at
C. up

6. A. those
B. blood
C. report

7. And ran a few yards before losing the

A. Bet. The police have launched a vigorous
B. Track
C. Game

8. A. search into the whole affair. They have
B. investigation
C. Campaign

9. A. given up the services of a forensic
B. requisitioned
C. Report

10. Expert

A. A fingerprint expert and a photographer.
B. An
C. Two

11. A. There are now fourteen horses at Raj
B. We
C. So

12. Bhavan

A. Who are kept in a large shed near the gate.
B. where
C. Which

ORDERING OF SENTENCE

Direction (Qs. 13-18): In the following items, each passage consists of six sentenced. The first and the sixth sentence are given in the beginning as S1 and S6. The middle four sentence in each have been removed and jumbled up. These are labeled P, Q, R and S. you are requested to find out the proper sequence of the four sentence and mark accordingly on the Answer Sheet.

13. S1 : There is no doubt the democracy is the best of the systems of government available to us.

S6 : It is this feature that puts democracy in a class by itself among political systems.
P : For another, even an individual can, through appeal to the judiciary, prevent the government from doing any injustice
Q : This means that, in a way, the people can exercise some control over the rulers even during their period of rule.
R : This right of the individual to secure Justice even against the powerful government is even more important than the right to vote.
S : For one thing, it permits, if necessary, a periodical change of those who govern the country.

The proper sequence should be :

A. RPQS
B. SQPR
C. PQRS
D. SRPQ

14. S1 : The authorities decided to streamline the procedure for admitting students to the new course.

S6 : The selection was made from this list in the order of priority.

P : Those who performed badly at the interview were eliminated.

Q : The candidates were first required to take a written test.

R : A list of successful candidates at the written test was prepared in the ascending order of total marks.

S : The top fifty among those who qualified were called for an interview.

The proper sequence should be :

A. QRSP
B. PRSQ
C. SRQP
D. RSQP

15. S1 : I was only 12 years of age then.

S6 : I needed real money.

P : But this was not enough even to buy comics or toys.

Q : I earned a couple of dollars by doing chores around the house.

R : I could never buy an airgun or a bike in this way.

S : I found a pressing need for money of my own.

The proper sequence should be :

A. SQPR
B. PSQR
C. SRPQ
D. QRPS

16. S1 : You ask me what I intend to do after graduation.

S6 : That would be a great relief to my parents.

P : My ambition is to becomes an engineer.

Q : I can complete this course in two years.

R : But I am not sure whether I shall get enough marks to qualify for admission to an engineering college

S : If I don’t take up engineering, I would like to do an M.Sc. in Physics.

The proper sequence should be :

A. PRSQ
B. SQPR
C. QSRP
D. RPQS

17. S1 : In 1974, Mark and Delia ventured into the Kalahari desert to study the ecology of the region.

S6 : She was moving towards them from about five metres away, her head swaying from side-to-side.

P : He slowly lifted his head and surveyed the scene.
Q : A soft groan startled mark.
R : They had driven north the evening before, trying to located the roars of a lion.
S : His breath caught-it was a huge lioness.

The proper sequence should be :

A. RQPS
B. RPQS
C. RQSP
D. QPSR

18. S1 : In almost all developing countries, estimates indicate that every six seconds, one child dies and another becomes disables as a consequence of diseases that can be prevented with immunization.

S6 : After each round of immunization during the IIP, all computerized data were brought up-to-date.
P : At the start of the programme, local Anganwadi centers for pre-school children operating as a part of the National Integrated Child Development Services Scheme were involved in conducting house-to-
Q : The experiment has been launched with the objective of intriguing new and efficient strategies for large-scale immunization programmes which, if successful, could be used as models for similar endeavors in the future.
R : Two areas were included in the IIP: Trilokpuri and Khanjawala.
S : Between December 1983 and March 1984, a large-scale experiment-the Intensive Immunization Programme (IIP)-was conducted in Delhi.

The proper sequence should be :

 A. QSPR
B. RPQS
C. PQRS
D. SQRP

ANTONYMS

Directions (Qs. 19-38): In this section, select the word or group of words that is most opposite in meaning to the bold word or group of words in the given sentence.

19. Poisonous gases emitted from factories contaminate the air we breathe in.

A. Sanctify
B. Invigorate
C. Taint
D. Purify

20. Reckless driving causes accidents.

A. Careful
B. Slow
C. Good
D. Correct

21. He often went to the theatre.

A. Seldom
B. Rarely
C. Sometimes
D. Occasionally

22. He is frugal in his spending.

A. Economical
B. Extravagant
C. Miserly
D. Greedy

23. The students expected eminent scientist to inaugurate the programme.

A. Illustrious
B. Notorious
C. Intelligent
D. Unknown

24. Some of their customs are barbarous.

A. Civilized
B. Modern
C. Polite
D. Praiseworthy

25. They are going to embark upon a mountaineering expedition.
A. Launch
B. Analyse
C. Break off
D. Conclude

26. There has been a gradual falling off in the quality of articles manufactured locally.

A. Shrinkage
B. Erosion
C. Impressed
D. Descent

27. He was deeply depressed over the news.

A. Satisfied
B. Elated
C. Impressed
D. Affected

28. Though he had lost the battle, he decided not to yield to the enemy.

A. Submit to
B. Persuade
C. Resist
D. Seek terms with

29. Because of the failure of the monsoon, there was paucity of food grains.

A. Overflow
B. Inflow
C. Plenty
D. Glut

30. The evidence against the accused is conclusive.

A. Powerful
B. Indecisive
C. Exclusive
D. Partial

Study Kit for Combined Defence Services Exam

Books for Combined Defence Services Exam

CDS Exam Solved Papers - 2012- I Paper I: General Knowledge

CDS Exam Solved Papers - 2012- I Paper I: General Knowledge

Paper I : General Knowledge

1. Before X-ray examination (coloured X-ray) of the stomach, patients are given suitable salt of barium because

A. Barium salts white in colour and this helps stomach to appear clearly
B. Barium is a good absorber of X-rays and helps stomach to appear clearly
C. Barium salts are easily available
D. Barium allows X-rays to pass through the stomach

2. The best colours for a Sun umbrella will be

A. Black on top and red on inside
B. Black on top and white on inside
C. Red on top and black on inside
D. White on top and black on inside

3. Which one among the following statements about globalization is not correct?

A. Advocates of globalization-argue that it will result I greater economic growth
B. Critics of globalization argue that it will result in greater in greater economic disparity
C. Advocates of globalization argue that it will result in cultural homogenization
D. Critics of globalization argue that it will result in cultural homogenization

4. Which one among the following statements is correct?

A. Due to border problem, India’s trade with China did not register a quantam jump between the years 2001-2010
B. The stand-off between India and China on Pakistan and Arunachal Pradesh did not let India’s trade to grow with China in the year 2001-2010 as much as it        grew in the year 1991-2001
C. While India-China trade has registered a quantaum jump between the year 2001-2010, both the countries have ensured parity of trade
D. In spite of border problems, India’s trade with China registered a quantam jump during the years 2001-2010

5. Which one among the following is a fundamental duty of citizens under the Constitution of India?

A. To provide friendly cooperation to the people of the neighboring countries
B. To protect monuments of national importance
C. To defend the country and render national service when called upon to do so
D. To know more and more about the history of India

6. Which among the following conditions are necessary for the issue of write of quo warrantor?

1. The office must be public and must be created by a Statue or by the Constitution itself.
2. The office must be substantive one and not merely the function or employment of a servant at the will and during the pleasure of another.
3. There has been a contravention of the Constitution or a Statute or Statutory Instrument, in appointing such person to that office.

Select the correct answer using the codes given below :

Codes :

A. 1 and 2 only
B. 1 and 3 only
C. 2 and 3 only
D. 1, 2 and 3

7. Which one among the following is the type of the Comoros Islands which lie in the Indian Ocean between Northern Madagascar and the African Coast?

A. Volcanic
B. Glacial eroded
C. Eolian deposited
D. Folded

8. The TEAM-9 initiative is a techno-economic cooperation venture between India and eight countries of

A. West Africa
B. East Africa
C. North- Africa
D. Central Africa\

9. Logically, what does a continually rising air pressure indicate?

A. Advent of unsettled and cloudy weather
B. Advent of a cyclone
C. Fine and settled weather
D. Fine and unsettled weather

10. The soil formed by wind dust in and around hot deserts is called

A. Salty soil
B. Loamy soil
C. Sandy soil
D. Loess soil

11. What are G. arobreum, G. herhaceum, G. hirsutum and G. barbadense, whose all the four species are cultivated in India?
(G = Gossypium)

A. Species of wool
B. Species of cotton
C. Species of silk
D. Species of jute

12. National income ignores

A. Sales of a firm
B. Sales of employees
C. Exports of the IT sector
D. Sales of land

13. The basis of European Union began with the signing of
 

A. Maastricht Treaty
B. Treaty of Paris
C. Treaty of Rome
D. Treaty of Lisbon

14. Which of the following measures should be taken when an economy is going through inflationary pressures?

1. The direct taxes should be increased.
2. The interest rate should be reduced.
3. The public spending should be increased.

Select the correct answer using the codes given below :

Codes :

A. 1 only
B. 2 only
C. 2 and 3
D. 1 and 2

15. Which of the following statements is/are correct?

1. If a country is experiencing increases in its per capita GDP, its GSP must necessarily be growing.
2. If a country is experiencing negative inflation , its GDP must be decreasing.

Select the correct answer using the codes given below:

Codes :

A. 1 only
B. 2 only
C. Both 1 and 2
D. Neither 1 nor 2

16. What is the major role of a green-house gas that contributes to temperature rise of the Earth’s surface?

A. Transparent to both incoming sunlight and outgoing infrared radiation
B. Stops both incoming sunlight and outgoing infrared radiation
C. Lets outgoing infrared radiation pass through but stops incoming sunlight
D. Lets incoming sunlight pass through but stops outgoing infrared radiation

17. Two identical piano wires have same fundamental frequency when kept under the same tension. What will happen if tension of one of the wires is slightly increased and both the wires are made to vibrate simultaneously?

A. Noise
B. Beats
C. Resonance
D. Non-linear effects

18. After rising a short distance the smooth column of smoke from a cigarette breaks up into an irregular and random pattern. In a similar fashion, a stream of fluid flowing past an obstacle breaks up into eddies and vortices which give the flow irregular velocity components transverse to the flow direction. Other examples include the wakes left in by whistling and by wind instruments. These examples are the result of

A. Laminar flow of air
B. Streamline flow of air
C. Turbulent flow of air
D. Viscous flow at low speed

19. Which one among the following correctly defines a unit magnetic pole in SI units? It is the pole which when placed in air at a distance of

A. 1 foot from an equal and a similar pole repels it with a force of 1 pound
B. 1 meter from an equal and similar pole repels it with a force of 1 Newton
C. 1 cm from an equal and a similar pole repels it with a force of 1 dyne
D. 1 meter from an equal and a similar pole repels it with a force of 1 Newton/m2

20. Lack of atmosphere around the Moon is due to

A. Low escape velocity of air molecule and low gravitational attraction
B. High escape velocity of air molecule and low gravitational attraction
C. Low gravitational attraction
D. High escape velocity of air molecule only

21. The element of a group in the periodic table

A. Have similar properties chemical
B. Have consecutive atomic numbers
C. Are isobars
D. Are isotopes

22. Which one among the following polymers is used for making bulletproof material?

A. Polyvinyl chloride
B. Polystyrene
C. Polyethylene
D. Polyamide

23. The pH of human blood is normally around

A. 4.5-5.5
B. 5.5-6.5
C. 7.5-8.0
D. 8.5-9.0

24. The pH of fresh groundwater slightly decreases upon exposure to air because

A. Carbon dioxide from air is dissolved in the water
B. Oxygen from air is dissolved in the water
C. The dissolved carbon dioxide of the groundwater escapes into air
D. The dissolved oxygen of the groundwater escapes into air

25. Who won the Wimbledon 2011 men’s singles title?

A. Novak Djokovic
B. Roger Federe
C. Rafael Nadal
D. Andy Roddick

26. Which among the following countries has recently decided to give a grant of US $ 1 billion for reconstruction of the Nalanda University?

A. USA
B. China
C. UK
D. Japan

27. Which one among the following is the fastest Indian Supercomputer recently developed by ISRO?

A. Aakash-A1
B. Saga-220
C. Jaguar-Cray
D. Tianhe-1A

28. The year 2011 has been celebrated by UNESOO and IUPAC as international Year of Chemistry to mark the 100 years of Nobel Prize winning by Madam Curie in Chemistry. In this context, which the following statements is/are correct?

1. The unified theme of the celebration is Chemistry-our life, our future.
2. The celebrations are made to mark the contributions of women to the development of science.

Select the correct answer using the codes given below :

Codes :

A. 1 only
B. 2 only
C. Both 1 and 2
D. Neither 1 nor 2

29. In February 2011, which among the following countries has elected Mr. Thein Sein, a military general-turned-civilian leader, as its firsts President?

A. Malaysia
B. Costa Rica
C. Thailand
D. Myanmar

30. Which one among the following vitamin is necessary for blood clothing?

A. Vitamin A
B. Vitamin D
C. Vitamin K
D. Vitamin C

Study Kit for Combined Defence Services Exam

Books for Combined Defence Services Exam

Public Administration Global Journals : Wilson's vision of Public Administration

Public Administration Global Journals : Wilson's vision of Public Administration

The Politics-Administration Dichotomy: Was Woodrow Wilson Misunderstood or Misquoted?

Politics-Administration Dichotomy

Historically, Woodrow Wilson was considered as the author of the separation of politics and administration in government (i.e., the politics-administration dichotomy). Thomas Woodrow Wilson (December 28, 1856 to February 3, 1924) was the 28th President of the United States (1913-1921). Among his main contributions, Wilson served as President of Princeton University from 1902 to 1910, as the Governor of New Jersey from 1911 to 1913 and was elected President as a Democrat in 1912 (Woodrow Wilson’s Biography, n.d.). Literature also shows that Woodrow Wilson was enormously successful as a doctorate graduate of Johns Hopkins University in 1883 and the author of The Study of Administration which he published in 1887. It is in this master piece that Wilson’s ideas and passion about the politics-administration dichotomy are succinctly presented. A careful analysis of literature shows that any understanding or misunderstanding of Wilson’s intentions about the politics-administration dichotomy is based on different interpretations or misinterpretations of this famous article.

Although the understanding and concept of the politics-administration dichotomy varies from writer to writer, depending on the historical period in which they write and the context they are writing about, the term dichotomy as applied here refers to the separation of politics and administration in policy design and implementation. The politics-administration dichotomy envisages public employees who can be said to be “impersonal” and “apolitical”, in the sense of having no political interests or political affiliations (Wilson, 1887). Writers and politicians who are also considered to be the pioneers in the advocacy of Wilson’s ideas of politics-administration dichotomy include Goodnow (1900) and Taylor (1912). Like Wilson, these two writers also see the attempt to create a bureaucracy with a distinct separation of the organisational and the political lives of bureaucrats o public administrators as underpinning this dichotomy.

The apolitical public employee is also what Max Weber envisioned when he developed his famous “bureaucratic” model in 1948. The ideal model of Max Weber’s bureaucratic structure was characterised by a clear division of labour, a well-defined authority hierarchy, high formalisation, career tracks for employees, and most importantly, “impersonality” and a total separation of members’ organisational lives from their personal lives and interests. Following are Wilson’s own views of the politics-administration dichotomy.

Wilson’s Own Writing

One of the main evidences about Wilson’s ideas of the politics-administration dichotomy is his own writing—The Study of Administration published in 1887. In his famous article, Wilson (1887, pp. 209-210) stated that: “The field of administration is… removed from the hurry and strife of politics; [and that] it at most points stands apart even from the debatable ground of constitutional study”. He went further to argue that “administration is a part of political life only as the methods of the counting-house are part of the life of society; only as machinery is part of the manufactured product”. Most importantly, Wilson viewed civil-service reform as “clearing the moral atmosphere of official life by establishing the sanctity of public office as a public trust, and, by making service unpartisan, it is opening the way for making it business-like”. Wilson’s argument that “administration lies outside the proper sphere of politics” and that “administrative questions are not political questions” is best expressed in his argument that “Politics is state activity in things great and universal, while administration, on the other hand, is the activity of the state in individual and small things” (Wilson, 1887, p. 210). According to Wilson, “Politics is thus the special province of the statesman, administration of the technical official. Policy does nothing without the aid of administration; but administration is not therefore politics”. Wilson stamped his authority on the authorship of the theory of politics-administration dichotomy in his statement that “we do not require German authority for this position; this discrimination between administration and politics is now, happily, too obvious to need further discussion”.

Wilson championed the ideas of the politics-administration dichotomy because, according to him, an “apolitical bureaucracy” was necessary in order to meet the economic and efficient implementation of the popular will (i.e., the will of the American people according to the Declaration of Rights and the Constitution) and the needs of a democracy operating in an increasingly industrialised economy (Wilson, 1887, 1941; Van Riper, 1990).

Past and Contemporary Writers

A number of classical and contemporary authors confirmed that Woodrow Wilson did indeed champion  the ideas of politics-administration dichotomy. The politics-administration dichotomy as envisioned in Wilson writing was also the central tenet of renowned classical writers and researchers such as Goodnow (1900) and Taylor (1912) among others. According to Gulick and Urwick (1937), Sayre (1958), and Campbell and Peters (2005), these classical authors conceptualised a government in which public officials (bureaucrats) had no affiliation to political parties and no aspiration to politics or to political life. Some contemporary authors who ascribed the ideas of politics-administration dichotomy to Wilson included Van Riper (1990) who argued that Wilson strove vigorously and self-confidently in the quest for “political neutrality”, coupled with technical competency in the civil service. However, while the classical literature (i.e., up to the early 1940s) seems to be unanimous on the author of the politics-administration dichotomy, some contemporary literatures claim that those who ascribe the politics-administration dichotomy to Wilson have misunderstood him.

Could Wilson be Misunderstood?

In spite of Wilson’s own writing which unambiguously presented the idea of politics-administration dichotomy (as presented above), a number of authors, such as Stillman (1973, p. 588), Van Riper (1984; as cited in O’Toole, 1987), and Rosenbloom (2008, pp. 57-60) as well as M. Dimock and D. Dimock offered critiques of those who attributed the politics-administration dichotomy concept to Wilson’s writings. Their claim was that Wilson did not set out to separate the public administration and politics functions, nor did he wish to exclude administrators from political affairs.
For example, Rosenbloom (2008, pp. 57-60) claimed that although Wilson’s article was very popular at the time, most authors who endorsed its thesis missed the point. As Rosenbloom put, its original intention was simply to advocate detaching partisan politics and patronage from sound public management. M. Dimock and D. Dimock also claimed that Wilson’s intention was not to advocate a “strict separation between politics and administration”.

In addition, despite a clear belief of the politics-administration dichotomy found in the summary of classical authors such as Wilson, Goodnow, Taylor, Gulick and Urwick among others discussed above in this paper, a number of authors such as Golembiewski (1977), and Van Riper (1990, pp. 209-210) argued that “these founding fathers of the field never advocated the dichotomy attributed to them”.
Finally, there are some authors who would neither ascribe the politics-administration dichotomy to Wilson nor publicly deny that he championed the idea of politics-administration dichotomy. These include O’Toole (1987, p. 18) who claimed that Wilson’s article was ambiguous, full of contradictions and poorly written to the point that it was difficult to be entirely certain of what message he was attempting to convey. There is also Svara (1999, p. 676) who rejected the idea of politics-administration dichotomy on the basis that those who represented the politics-administration dichotomy model as historically significant, or as conceptually and empirically faulty, have missed two fundamental points. The first is that classical theorists such as Wilson were not explicitly suggesting or advocating the separation between politics and public administration, a concept that the literature of the 1940s and 1950s initially explored. The second, as Svara (2001, pp. 176-183) continued to argue, is that the term “dichotomy” itself was rarely used before the 1950s. As such, Svara (2001) concluded, the “founders” of the field, who were supposed to have invented the politics-administration dichotomy model, could not have used this particular term in their writings (Svara, 1999, p. 676; 2001, p. 176). The argument about the beginning of the use of the term “dichotomy” is not the subject of this research. However, it was inconceivable to admit that the meaning and the ideas represented by the term

“politics-administration dichotomy” were not what the literature of the early 1940s, including Wilson’s famous article—The Study of Administration quoted above, championed. While Svara might be right about the non-existence of the term “dichotomy” before the 1950s, its proper meanings such as: “discrimination between administration and politics, administration which lies outside the proper sphere of politics…” are clearly used in Wilson’s article which summarised in this paper.

In the absence of literature of the early 1900s and late 1950 which found Wilson’s writing confusing, and lack of evidence about Wilson’s defending himself against writers of his era who ascribed the idea of politics-administration dichotomy to him, Svara’s argument that the politics-administration dichotomy concept “was rather a poorly grounded characterization of the early literature that took hold in the late 1950s” (Svara,
1999, p. 676) cannot hold water.

Public Administration Study Kit For Mains Examination

Public Administration Mains Test Series 2017-18

CDS Exam Solved Papers - 2011- II Paper I: General Knowledge

CDS Exam Solved Papers - 2011- II Paper I: General Knowledge

Directions: The following Seven (07) items consist of two statements, statement I and statement’ II. ‘ You are to examine these two statements carefully and select the answers to these items using the code given below

Code:

(a) Both the statements are individually true and statement II is the correct explanation of statement I
(b) Both the statements are individually true but statement II is not the correct explanation of statement I
(c) Statement I is true but statement II is false
(d) Statement I is false but statement II is true

1. Statement I: El Nino is a temperature rising phenomenon over the Pacific Ocean and usually causes dry mon­soon in South Asia.
Statement II : Tsunamis are usually not noticed as the massive ocean waves move silently but assume destructive form as these travel. through shallow waters of continental shelves.

2. Statement I: Anticyclone, which is a high pressure wind system, does not bring about significant change in weather condition.
Statement II: The outward movement of wind from the high pressure centre keeps limited scope for weather disturbance.

3. Statement I: Annie Besant worked together with the Congress and the Muslim League during the Home Rule Movement.
Statement II: Annie Besant felt that this was necessary to get the support of the masses for the Home Rule Movement.

4. Statement I: Deficit financing does not lead to inflation if adopted in small doses.
Statement II: Deficit financing is an often used tool for financing budgetary deficits.

5. Statement I: Oxides of sulfur and nitrogen present in high concentration in air are dissolved in rain drops.
Statement II: Oxyacids of sulfur and nitrogen make rain water acidic.

6. Statement I: On mixing with water, Plaster of Paris hardens.
Statement II: By combining with water, Plaster of Paris is converted into Gypsum.

7. Statement I: All liquids are conductor’s of electricity.
Statement II: Under the condition of low pressure and high voltage, liquids can be made conducting.

8. Name the Continents that form a mirror image of each other

(a) North America and South America
(b) Asia and Africa
(c) Africa and South America
(d) Europe and Asia

9. How many kilometres are represented by 1 ° of latitude ?

(a) 321 km
(b) 211 km
(c) 111 km
(d) 91 km

10. Which one among the following rivers does not flow into the Bay of Bengal ?

(a) Mahanadi
(b) Cauvery
(c) Tapti
(d) Godavari

11. Which one among the following cities never get the vertical rays of the Sun all through the year ?

(a) Chennai
(b) Mumbai
(c) Kolkata
(d) Srinagar

12. Climate change resulting in the rise of temperature may benefit which of the countries/regions ?

(a) South Africa
(b) East Indies islands comprising of Java, Sumatra and Borneo
(c) The western coasts of South America
(d) Russia and Northern Europe

13. Which of the following statements regarding ozone layer within the atinosphere is/are correct ?

1. It absorbs most of the ultraviolet radiation found in the Sun’s rays.
2. Chlorofluorocarbons are serious threat to the ozone layer.

Select the correct answer using the code given below

(a) 1 only
(b) 2 only
(c) Both 1 and 2
(d) Neither 1 nor 2

14. When we consider 15° meridian on a world map or globe and count them in an eastward direction starting with Greenwich meridian (0°), we find that the time of this meridian is

(a) same as Greenwich
(b) 1 hour fast
(c) 1 hour slow
(d) 12 hours fast

15. What is the target of the Millennium Development Goal of the United Nations with respect to Universal Primary Education ?

(a) All children both boys and girls would complete a full course primary schooling by 2015
(b) All children both boys and girls are to be literate by 2050.
(c) All boys should go to school to complete a full course of primary schooling by 2025
(d) All girls should be enrolled in , primary school by 2015

16. The broken hills famous for zinc and lead are located in

(a) Turkey
(b) France
(c) Germany
(d) Australia

17. Kanha National Park belongs to which one among the following biogeographical areas in the world ?

(a) Tropical Sub-humid Forests
(b) Tropical Humid Forests
(c) Tropical Dry Forests
(d) Tropical Moist Forests

18. ‘El Nino’ associated with the formation of the South West Monsoon of India is

(a) an abnormally warm ocean current
(b) a periodic warm air-mass
(c) a periodic warm wind
(d) a periodic low pressure centre

19. The thermal equator is found

(a) at the equator
(b) south of the geographical equator
(c) north of the geographical equator
(d) at the tropic of cancer

20. A nautical mile is equal to

(a) 5060 feet
(b) 5280 feet
(c) 6060 feet
(d) 6080 feet

21. Horse latitudes lie within the atmospheric pressure belts of

(a) Polar high
(b) Equatorial low
(c) Sub-tropical high
(d) Sub-polar low

22. The warm and dry winds that blow down the steep valleys in Japan are called

(a) Zonda
(b) Yaino
(c) Tramontane
(d) Santa Ana

23. Which off the following statements regarding red soils of India is/are correct?

1. The colour of the soil is red due to ferric oxide content.
2. Red soils are rich in lime. humour and potash.
3. They are porous and have friable structure.

Select the correct answer using the code aiven below

(a) 1 only
(b) 1 and 3 only
(c) 2 and 3 only
(d) 1, 2 and 3

24. According to the Census 2011, which one among the following Union Territories has the least population ?

(a) Daman and Diu
(b) Dadra and Nagar Haveli
(c) Lakshadweep
(d) Puducherry

25. Arrange the following oil refineries of India from west to east

1. Koyali
2. Bongaigaon
3. Mathura
4. Haldia

Select the correct answer using the code given below:

(a) 1 - 2 - 3 - 4
(b) 1 - 3 - 4 - 2
(c) 3 - 1 - 2 - 4
(d) 2 - 4 - 3 - 1

26. Which one among the following creeks is not associated with the state of Gujarat ?

(a) Kori creek
(b) Godai creek
(c) Kajhar creek
(d) Sir creek

27. Match List I with List II and select the correct answer using the code given below. the Lists

List I                                     List II
(Local wind)                       (Area of prevalence)

A. Chinook                         1. North African desert

B. Foehn                             2. Rocky mountain slopes of the USA

C. Sirocco                          3. Northern slopes of Alps

D. Mistral                           4. Southern slopes of Alps

Code:

    A B C D

(a) 2 3 1 4
(b) 2 1 3 4
(c) 4 1 3 2
(d) 4 3 1 2

28. Which one of the following is not a formal agency of social control ?

(a) Courts
(b) Police
(c) Family
(d) Teachers

29. The term used to describe the process by which an outsider, immigrant or subordinate group becomes indistin-guishably integrated into the dominant host society, is known as

(a) Accommodation
(b) Cultural impcrialism
(c) Acculturation (d) Adaptation

30. Which among the following is/are example/examples of youth unrest in India?

1. Naxalite movement
2. Anti-foreigners movement in Assam
3. Anti-Mandal Commission agitation

Select the correct answer using the code given below

(a) 1, 2 and 3
(b) 2 and 3 only
(c) 2 only
(d) 3 only

Study Kit for Combined Defence Services Exam

Books for Combined Defence Services Exam

CAPF-AC Solved Papers : 2015

CAPF-AC Solved Papers : 2015

1. The setting up of the Inter-State Council in 1990 was meant to

(a) substitute the National Development Council
(b) strengthen the federal provisions of the Constitution
(c) be an institutional interface between the· Judiciary and the Government
(d) provide membership to local customary bodies

2. Which one of the following is not a Central Paramilitary Force under the Ministry of Home Affairs?

(a) Central Industrial Security Force
(b) Central Reserve Police Force
(c) Ladakh Scouts
(d) Border Security Force

3. The following item consists of two Statements I and II. Examine these two statements carefully and select the correct answer using the code given below.

Statement I: India's share of world manufacturing output steadily declined during the 19th and the 20th centuries.
Statement II: There was no absolute decline in production in India during the colonial rule.

Codes

(a) Both the statements are individually true and Statement II is the correct explanation of Statement I
(b) Both the statements are individually true but Statement II .is not the correct explanation of Statement I
(c) Statement I is true but Statement II is false
(d) Statement I is false but Statement II is true

4. The Comptroller and Auditor-General of India. can be removed from office only by

(a) President on the advice of the Union Cabinet
(b) Chief Justice of the Supreme Court
(c) President of India after an address in both Houses of Parliament
(d) President on the advice of Chief Justice of India

5. Which one of the following is under the control of the Ministry of Defence?

(a) lndo-Tibetan Border Police
(b) Rashtriya Rifles
(c) National. Security Guard
(d) Sashastra Seema Bal

6. Which of the following statements with regard to construction of railways in colonial India by the British companies is/are not true?

1. The companies were guaranteed a return of 5% on their investment by Government of India.
2. The railways were to be managed mainly by the Government.'
3. There was no system bf preferential freight charges.
4. The companies were to get the land free from the Government.

Select the correct answer using the code given below.

(a) 1 and 3
(b) 2 and 3
(c) Only 4
(d) 2.3 and 4

7. According to Granville Austin, which of the following was/were the most. significant contributions of India to constitution-making?

(a) Majority Rule and Minority Rights
(b) Consensus and Accommodation
(c) Social revolution and Political Freedom
(d) Asymmetrical Federal Structure

8. Jammu and Kashmir Light Infantry is

(a) a Central Paramilitary Force .
(b) a force maintained by the state of Jammu and Kashmir
(c) a regiment of the Indian Army
(d) None of the above

9. Which of the following represents the main phases of the Quit India Movement?"

1. Strikes, boycotts and picketinq in urban centres.
2. Widespread attacks in rural areas on the many symbols and means of colonial rule and authority. e.g., railways, telegraph lines and Government buildings.
3. Formation of Home Rule leagues.
4. The Karnataka method.

Select the correct answer using the code given below.

(a) 1 and 2
(b) 2, 3 and 4
(c) 1, 2 and 4
(d) 1 and 4

10. The Special Provisions in Article 371G of the Constitution of India relate to the state of

(a) Jammu and Kashmir
(b) Jharkhand
(c) Mizoram
(d) Nagaland

11. Who among the following resigned from his post in the year 2014?

(a) Chief of the Air Staff
(b) Chief of the Naval Staff
(c) Director General Coast Guard
(d) Chief of the Integrated Defence Staff

12. Which of the following was/were not part/parts of the RIN Mutiny?

1. Indian National Army
2. HMIS Talwar
3. Hunger strike by naval ratings
4. Lord Irwin

Select the correct answer using the code given below.

(a) 1 and 2
(b) 2 and 3
(c) 1 and 4
(d) Only 4

13. The Locus Standi rule to move the court was liberalized by the case of

(a) SR Bornrnai vs Union of India
(b) Minerva Mills vs Union of India
(c) SP Gupta vs Union of India
(d) Kesavananda Bharati vsState of Kerala

14. Who heads the Defence Acquisition Council?

(a) Defence Minister
(b) Defence Secretary
(c) Chief of the Integrated Defence Staff
(d) Director General (Acquisition)

15. Bal Gangadhar Tilak was associated with

1. Poona Sarvajanik Sabha
2. The Age of Consent Bill
3. Gaurakshini Sabha
4. Atmiya Sabha

Select the correct answer "using the code given below.

(a) 1 and 2
(b) 1, 2 and 4
(c) 3 and 4
(d) 2 and 4

16. The provision under Article 350A of the Constitution of India relates to the

(a) right of any section of the citizens to preserve its distinct Iqnguage and culture
(b) right of the Sikh community to carrying and wearing of kirpans
(c) provision for providing facilities for instruction in mother-tongue at primary stage
(d) freedom of Minority-managed educational institutions from discrimination in the matter of receiving aid from the State

17. The equivalent rank of Commodore of Indian Navy in the Indian Army is'

(a) Brigadier
(b) Lieutenant Colonel
(c) Colonel
(d) Major General

18. The proposals of the Cabinet Mission did not include .

1. a three-tier structure for the Union of India, with both the Provinces and the Princely States.
2. formation of a Constituent Assembly.
3. creation of a sovereign Pakistan with six Muslim-majority provinces.

Select the correct answer using the code given below.

(a) 1 and 3
(b) Only 3
(c) Only 2
(d) 1 and 2

19. Which of the following are provided in India by the Right to Education Act?

1. Right of children to free and compulsory education till completion' of elementary education in a neighbourhood school.
2. Prohibition of deployment of teachers for non-educational works, other than decennial census, elections to local authority like State Legislatures, Parliament and disaster relief.
3. Right' of minorities to establish and administer education institution.
4. No citizen shall be denied admission into any educational institution maintained by the State or i:eceiving aid out of State funds on grounds only of religion, race, caste, language or any of them.

Select the correct answer using the code given below.

(a) 1, 2 and 4
(b) 1 and 3
(c) 2 and 4
(d) 1 and 2

20. INS Vikramaditya is the name of

(a) an aircraft carrier
(b) a nuclear submarine
(c) a memorial for the sailors
(d) a naval hospital

केन्द्रीय सशस्त्र पुलिस बल (सहायक कमांडेंट) के लिये स्टडी किट

Study Kit for Central Armed Police Forces

UPSC IAS PRELIMS Exams 2017 Results declared

IAS EXAM

(BREAKING NEWS) UPSC IAS PRELIMS Exams 2017 Results declared

BREAKING : UPSC IAS PRE 2017 Results declared. 13600+ Candidates clear exam.

CIVIL SERVICES (PRELIMINARY) EXMINATION, 2017

On the basis of the result of the Civil Services (Preliminary) Examination, 2017 held on 18/06/2017, the candidates with the following Roll Numbers have qualified for admission to the Civil Services (Main) Examination, 2017.

The candidature of these candidates is provisional. In accordance with the Rules of the Examination, all these candidates have to apply again in the Detailed Application Form, DAF (CSM), for Civil Services (Main) Examination, 2017, which would be available on the website of the Union Public Service Commission www.upsc.gov.in. All the qualified candidates are advised to fill up the DAF (CSM) online and submit the same ONLINE for admission to the Civil Services (Main) Examination, 2017 to be held from Saturday, the 28th October, 2017. The DAF (CSM) will be available on the website of the Commission from 17th August, 2017 to 31st August, 2017 till 6.00 P.M. Important instructions for filling up of the DAF (CSM) an for submitting the completely filled application form ONLINE, to the Commission, would also be available on the website. The candidates who have been declared successful have to first get themselves registered on the relevant page of the website before filling up the ONLINE DAF. The qualified candidates are further advised to refer to the Rules of the Civil Services Examination, 2017 published in the Gazette of India (Extraordinary) of Department of Personnel and Training Notification dated 22.02.2017.

It may be noted that mere submission of application form DAF(CSM), either online or the printed copy thereof, does not, ipso facto, confer upon the candidates any right for admission to the Main Examination. The e-Admit card along with the time table of the Main Examination will be uploaded on the Commission’s Website to the eligible candidates around 2 weeks before the commencement of the examination. Changes, if any, in the postal address or email address or mobile number after submission of the DAF (CSM) may be communicated to the Commission at once. 

Candidates are also informed that marks, cut off marks and answer keys of screening test held through CS (P) Examination, 2017 will be uploaded on the Commission website i.e., www.upsc.gov.in only after the entire process of CS(M) Examination, 2017 is over i.e. after the declaration of final result of Civil Services Examination, 2017. The Union Public Service Commission have a Facilitation Counter near the Examination Hall Building in its Campus at Dholpur House, Shahjahan Road, New Delhi.

Candidates may obtain any information/clarification regarding their result of the above mentioned examination on all working days between 10.00 AM to 5.00 PM, in person or on Tel. No. 011-23385271, 011-23098543 or 011-23381125 from this Facilitation Counter.

Candidates can also obtain information regarding their result by accessing Union Public Service Commission Website www.upsc.gov.in

The result of Roll Number 1000001 has been withheld as the issue regarding his candidature is subjudice and the result of Roll No.0178081 is withheld as her candidature is Provisional till the finalization of her disciplinary case.

Click Here For Official Result

Courtesy : UPSC
 

NDA/NA Exam Solved Papers - 2011- I - Paper I: General Ability Test (Part-D)

NDA/NA Exam Solved Papers - 2011- I - Paper I: General Ability Test (Part-D)

Paper I : General Ability Test

121. The Clean Development Mechanism (CDM), a mechanism to reduce green-house gas emission as per Kyoto Protocol implies that

(a) industrial countries receive carbon credits by funding carbon saving projects in another relatively affluent nation
(b) industrial countries reduce their carbon emission by using environment friendly technology in production
(c) developed countries invest in carbon reduction in developing countries and receive carbon credit in return
(d) developed nations purchase carbon credit from other nations

122. Match List-I with List-II and select the correct answer using the code given below the Lists :

List-I                         List-II
(Joint exercise)         (Type)

(a) Indra                 1. Joint anti-terrorism exercise
(b) Cope India        2. Joint air force exercise
(c) Ind-Indo Sorpat 3. Coordinated mari- time patrolling
(d) Yudhabhyas       4. Joint army exercise

Code :

    A B C D
(a) 1 2 3 4
(b) 1 3 2 4
(c) 4 3 2 1
(d) 4 2 3 1

123. Match List-I with List-II and select the correct answer using the code given below ‘the Lists :

List-I                             List-II
(Ship)                             (Type)

(a) INS Arihant         1. Aircraft Carrier
(b) INS Vikrant         2. Landing Platform Dock
(c) INS Shiwalik       3. Nuclear Submarine
(d) INS Airavat         4. Stealth Frigate

Code :

    A B C D
(a) 3 4 1 2
(b) 3 1 4 2
(c) 2 1 4 3
(d) 2 4 1 3

124. ‘Milan’ is ?

1. an annual gathering of navies. of Indian Ocean countries.
2. conducted at Chennai.
3. a joint antiterrorism exercise.
4. aimed at enhancing inter-operability among navies.

Select the correct answer using the code given below :

(a) 1 only
(b) 2 and 3
(c) 1 and 4
(d) 3 and 4

125. Consider the following pairs :

1. Pinaka : 1. Multi Barrel  Rocket Launcher System
2. Nag : 2. Anti Tank Missile System
3. Lakshya : 3. Unmanned Aerial Vehicle

Which of the pairs given above is/are correctly matched ?

(a) 1 only
(b) 1 and 2 only
(c) 2 and 3 only
(d) 1, 2 and 3

126. Which one among the following is the unit raised to protect the naval assets ?

(a) Sagar Rakshak Bat
(b) Sagar Suraksha Bat
(c) Sagar Prahari Bat
(d) Sagar Nigrani Bat

127. Which one among the following films won the Nargis Dutt award for best feature film on national integration in the 57th National Film Award in the year 2010 ?

(a) Well Done Abba
(b) Lahore
(c) Dilli-6
(d) Paa

128. Consider the following statements with regard to the Renewable Energy Certificate (REC) Mechanism launched in November 2010 by the Government of India :

1. It enables the obligated entities to meet their renewable purchase obligation
2. It is one of the pioneering efforts in any developing country for mainstreaming the renewable energy generation through market mechanism

Which of the statements given above is/are correct ?

(a) 1 only
(b) 2 only
(c) Both 1 and 2
(d) Neither 1 nor 2

129. Consider the following statements with regard to the Global Hunger Index :

1. India ranks within the top ten among the developing countries in the Global Hunger Index 2010
2. According to the Global Hunger Index 2010 the proportion of undernourished in India is decreasing
3. Normally economic progress of a country measured by its gross national income and the hunger level are inversely correlated

Which of the statements given above is/are correct ?

(a) 1, 2 and 3
(b) 2 and 3 only
(c) 1 and 2 only
(d) 3 only

Directions: The following Six (06) items consist of two statements, statement / and statement II. You are to examine these two statements carefully and select the answers to these items using the code given below:

Code :

A. Both the statements are individually true and statement II is the correct explanation of statement I
B. Both the statements are individually true but statement II is not the correct explanation of statement I
C. Statement I is true but Statement II is false
D. Statement I is false but statement II is true

130. Statement I. A myopic person is advised to use concave lens
Statement II. The eye lens of a myopic person focuses the parallel rays coming from distant objects in front of the retina

131. Statement I. Oxidation in our body cell releases dangerous free radicals
Statement II. Our body itself produces anti-oxidants to neutralize harmful free radicals

132. Statement I. Decay and disintegration of rocks in situ is called weathering
Statement II. Mechanical weathering is mainly caused by temperature variation

133. Statement I. The semi-arid tracts of India stretching from eastern “ Rajasthan in the north to south central Tamil Nadu are agriculturally less productive
Statement II. The semi-arid tracts are homeland to a large number of central Indian Scheduled Tribe population

134. Statement I. The Mediterranean climate is highly suitable for fruit production
Statement II. Cool and moist winters in Mediterranean regions enable ample’ production of fruits.

135. Statement I. In the northern hemisphere the ocean currents flowing from equator towards the north pole and from pole’ towards the equator are deflected to their right
Statement II. This happens due to rotation of the Earth on the axis from west to east

136. A bee-sting leaves an acid which causes pain and irritation. The injected acid is :

(a) acetic acid
(b) sulphuric acid
(c) citric acid
(d) methanoic acid

137. Iron nails are dipped into blue copper sulphate solution. After some time iron nails are :

(a) dissolved and blue colour is discharged
(b) dissolved but blue colour is not discharged
(c) not dissolved but blue colour is not discharged
(d) not disssolved but blue colour is discharged

138. A student by chance mixed acetone with alcohol. This mixture of acetone and alcohol can be separated by :

(a) filtration
(b) separating funnel
(c) fractional crystallization
(d) fractional distillation

139. Silver ware turns black after a period of time due to formation of :

(a) nitrate coating on silver
(b) sulphide coating on silver
(c) chloride coating one silver
(d) oxide coating on silver

140. Which of the statements given below is/are correct

Permanent hardness of water is due to the presence of soluble :

1. chloride of calcium
2. bicarbonate of calcium
3. sulphate of magnesium
4. bicarbonate of magnesium

Select the correct answer using the code given below :

(a) 1 only
(b) 1 and 3 only
(c) 2 and 4
(d) 1, 2 and 3

141. Which of the following statements about diamond are correct ?

1. It is used as a gem in jewellery because of its ability to reflect light
2. It is a good conductor of electricity
3. It is used for cutting glass, marble stones and other hard materials
4. It is used for drilling of rocks

Select the correct answer using the code given below :

(a) 1, 3 and 4
(b) 2, 3 and 4
(c) 1, 2 and 3
(d) 2 and 4 only

142. ‘Which one among the following methods is not effective in removing arsenic from contaminated ground water?

(a) Boiling
(b) Reverse osmosis
(c) Ion exchange
(d) Coagulation-adsorption

143. Bronze is often used to make statues and medals whereas brass is used in making utensils, scientific apparatus and cartidges. Both brass and bronze are copper containing alloys, yet they differ in their chemical composition for additionally containing :

(a) Zinc in brass and tin in bronze
(b) Chromium in brass and nickel in bronze
(c) Nickel in brass and tin in bronze
(d) Iron in brass and nickel in bronze

144. Which of the statements about glass are correct ?

1. Glass is a super-cooled liquid having infinite viscosity
2. Violet coloured glass is obtained by adding MnO2
3. Glass is a man made silicate
4. Glass is -a crystalline substance

Select the correct answer using the code given below :

(a) 1, 2 and 4
(b) 2, 3 and 4
(c) 1, 2 and 3
(d) 1 and 3 only

145. Sandalwood tree is considered a :

(a) total root parasite
(b) total stem parasite
(c) stem parasite
(d) partial ‘root parasite

146. Which one among the following statements about stomach is not correct ?

(a) Stomach acts as a temporary reservoir
(b) Stomach mixes food with gastric juice
(c) Stomach secretes lipase and amylase in gastric juice
(d) Rate of stomach emptying depends on the type of food

147. Which one of the following organs breaks fat to produce cholesterol ?

(a) Intestine
(b) Liver
(c) Lungs
(d) Kidneys

148. Consider the following statements about comets :

1. Most comets have elongated elliptical orbits that take them close to the Sun for a part of their orbit, and then out into the further reaches of the Solar System for the remainder
2. If a comet is travelling fast enough, it may leave the Solar System

Which of the statements given above is/are correct ?

(a) 1 only
(b) 2 only
(c) Both 1 and 2
(d) Neither 1 nor 2

149. An individual whose blood type is B may in an emergency donate blood to a person whose blood type is :

(a) B or A
(b) AB or A
(c) A or O
(d) AB or B

150. Which of the following statements is/are correct ?

1. In comparison to the Jupiter, planet Earth displays eclipses more frequently
2. On Mars only partial solar eclipses are possible

Select the correct answer using the code given below :

(a) 1 only
(b) 2 only
(c) Both 1 and 2
(d) Neither 1 nor 2

121. (c) 122. (a) 123. (b) 124. (c) 125. (d) 126. (b) 127. (c) 128. (c) 129. (d) 130. (a)
131. (b) 132. (c) 133. (b) 134. (a) 135. (a) 136. (d) 137. (a) 138. (d) 139. (d) 140. (b)
141. (a) 142. (a) 143. (a) 144. (c) 145. (d) 146. (c) 147. (b) 148. (c) 149. (d) 150. (c)

Study Material for UPSC NDA/NA Examination

Books for UPSC NDA/NA Examination

Courtesy : UPSC

 

NDA/NA Exam Solved Papers - 2011- I - Paper I: General Ability Test (Part-C)

NDA/NA Exam Solved Papers - 2011- I - Paper I: General Ability Test (Part-C)

Paper I : General Ability Test

81. Suppose you are standing 1 m in front of a plane mirror. What should be the minimum vertical size of the mirror so that you can see your full image in it?

(a) 0.50 m
(b) 2 m
(c) half of your height
(d) twie your height.

82. Light travels slower in glass than in air because :

(a) refractive index of air is less than that of glass.
(b) refractive index of air is greater than that of glass.
(c) density of glass is greater than that of air.
(d) density of glass is less than that of air.

83. The lines of force of a uniform magnetic field

(a) must be convergent
(b) must be divergent
(c) must be parallel to each other
(d) intersect.

84. A Jet engine works on the principle of conservation of :

(a) linear momentum
(b) angular momentum
(c) energy
(d) mass

85. The surface temperature of the Sun is nearly

(a) 2000 K
(b) 4000 K
(c) 6000 K
(d) 8000 K

86. Venkataraman Ramakrishnan was jointly awarded Nobel Prize in Chemistry in the year 2009 for the :

(a) theory of electron transfer.
(b) studies of the structure and function of the ribosome.
(c) palladium catalyzed cross couplings in organic synthesis.
(d) work in the area of olefin metathesis.

87. Which one among the following statements regarding cell is not correct?

(a) Shape and size of cells are related to specific function
(b) Some cells have changing shapes
(c) Each cell has its own capacity to perform
(d) Same type of cells are present in all body tissues

88. Which one among the following Indian scientists proposed a theory for long distance transport of water in plants?

(a) J.C. Bose
(b) Birbal Sahni
(c) P. Maheshwari
(d) N.S. Parihar

89. To a perpendicular to the plane of ecliptic, the Earth’s axis of rotation makes an angle of 23 degrees. Had this angle been 0 degree, which one among the following would result?

(a) There would have been no seasons
(b) The length of day and night would have been the same throughout the year
(c) The length of day and night would have been the same all over the earth
(d) All of the above

90. Jet streams are usually found in the :

(a) Ozonosphere
(b) Mesosphere
(c) Tropopause
(d) Ionosphere

91. There exceptionally high and low tides that occur at the time of the new moon or the full moon when the Sun, ‘the Moon and the Earth are approximately aligned is called:

(a) Spring
(b) Fall
(c) Neap
(d) Diurnal

92. What is the general direction of cyclones formed in the Bay of Bengal?

(a) East to west
(b) West to east
(c) West to south
(d) North to south

93. The Narmada river in the Peninsular plateau flows westward with a remarkably straight channel. It is because the :

(a) slope gradient in this part controls the river channel pattern.
(b) river carries a huge amount of water which has created a straight channel course.
(c) river forms the boundary between the Central Highlands and the Deccan Plateau.
(d) river flows through the trough of a rift valley inclined westward.

94. No trees are found in Tundra biome near polar region of northern hemisphere. This is due to :

(a) snowfall inhibits plant respiration.
(b) frozen ice beneath the surface soil (permafrost) restricts roots growth.
(c) less wind movement and inadequate sunlight.
(d) low temperature which restricts development of reproductive organs.

95. Why the summer monsoon winds blow from south-western direction in the northern hemisphere?

(a) The general direction of wind from the Indian Ocean is south-western
(b) The presence of the doldrums around the Equator
(c) The low pressure condition in north-west India
(d) Due to the effect of Coriolis force

96. If the Earth’s axis were perpendicular to the plane of its orbit, which one among the following would not have happened?

(a) The North Pole will always lie in dark
(b) Days and nights would be equal throughout the year
(c) No change of seasons will take place
(d) The sun will be perpendicular to the Equator

97. Match List-I with List-II and select the correct answer using the code given below the Lists:

98. Which one among the following is the best reason for the marked increase in the agricultural production in India in the past decades?

(a) Increase in the area under cultivation
(b) Conversion of barren land into agricultural land
(c) Use of improved agricultural methods and technologies
(d) Priority status given by the successive governments to atricultural sector over the industry sector

99. Which of the following statements regarding the Deccan Traps is/are correct?

1. Intense volcanic activity in the form of fissure eruption took place towards the end of Cretaceous period
2. The volcanic lava spread out in horizontal sheets
3. The regular soil found here is rich in nitrogen

Select the correct answer using the code given below :

(a) 1 and 2 only
(b) 1, 2 and 3
(c) 3 only
(d) 1 only

100. Consider the following statements :

1. The Himalayan vegetation varies according to both altitude and climatic conditions.
2. There are mainly two types of tropical forests that are found in the Himalayas the tropical rainforests and the tropical deciduous forests

Which of the statements given above is/are correct ?

(a) 1 only
(b) 2 only
(c) Both 1 and 2
(d) Neither 1 nor 2

101. If the electrical resistance of a typical substance suddenly drops to zero, then the substance is called :

(a) super conductor
(b) semiconductor
(c) conductor
(d) insulator

102. A spherical air bubble is embedded in a piece of glass. For a ray of light passing through the bubble, it behaves like a :

(a) converging lens
(b) diverging lens
(c) plano-converging lens
(d) plano-diverging lens

103. ‘The stars seem to be higher on the sky than they actually are’. This can be explained by:

(a) atmospheric refraction
(b) dispersion of light
(c) total internal reflection
(d) diffraction of light

104. Which one among the following is not a source of renewable energy?

(a) Hydroelectricity
(b) Solar energy
(c) Fuel cell
(d) Wind energy

105. A liquid rises to a certain length in a capillary tube. The tube is inclined to an angle of 45°. The length of the liquid column will :

(a) increase
(b) decrease
(c) remain unchanged
(d) first decrease and then increase

106. Mass of B is four times that of A, B moves with a velocity half that of A. Then B has :

(a) kinetic energy equal to that of A
(b) half the kinetic energy of A
(c) twice the kinetic energy of A
(d) kinetic energy one-fourth of A

107. In a pressure cooker cooking is faster because the increase in vapour pressure :

(a) increase the specific heat
(b) decreases the specific heat
(c) decreases the boiling point
(d) increases the boiling point

108. Magnets attract magnetic substances such as iron,. nickel, cobalt etc. They can also repel :

(a) paramagnetic substances
(b) ferromagnetic substances
(c) diamagnetic substances
(d) non-magnetic substances

109. When a ray of light is going from one medium to another, its :

(a) wavelength remains same
(b) frequency remains same
(c) frequency increases
(d) wavelength increases

110. Which one among the following is the main ingredient in cement?

(a) Gypsum
(b) Lime stone
(c) Clay
(d) Ash

111. Glass is actually :

(a) a crystalline solid
(b) an ionic solid
(c) an elastic solid
(d) a vitrified liquid

112. Solutions in test tubes containing H20 and aqueous NaOH can be differentiated with the help of :

(a) red litmus
(b) blue litmus
(c) Na2CO3
(d) HCl (aqueous)

113. A student heated some sulphur in a spatula and collected the gas ‘X’. Which one among the following is correct about ‘X’?

(a) X is SO2 and it turns moist litmus to blue
(b) X is SO3 and it turns moist litmus to red
(c) X is SO2 and it turns moist litmus to red
(d) X is SO3 and it turns dry litmus to blue

114. Human stomach produces acid ‘X’ which helps in digestion of food. Acid ‘X’ is :

(a) acetic acid
(b) methanoic acid
(c) hydrochloric acid
(d) citric acid

115. When concentrated H2S04 spilts on the surface, it should be immediately cleaned :

(a) with a piece of cloth
(b) by adding cold water
(c) by adding solid Na2CO3
(d) by adding solid BaCl2

116. India by becoming a full-fledged member of the Financial Action Task Force :

(a) will be able to access information on suspect’ accounts in nations such as Switzerland and UK
(b) will play an important role in law enforcement matters, investigations at an international level
(c) has also become a member of Organization for Economic Co-operation and Development
(d) all of the above

117. Consider the following declaration and identify the person who made it using the code given below :

‘The time has come when badges of honour make out shame glaring in their incongruous context of humiliation, and, I, for my part, wish to stand shorn of all special distinction, by the side of my countrymen who, for their so-called insignificance are liable to suffer degradation not fit for human beings’.

Code :

(a) Mahatma Gandhi
(b) Jawaharlal Nehru
(c) Dada Bhai Naoroji
(d) Rabindranath Tagore

118. Identify, using the code given below, the nationalist leader who was sentenced in the words’ outlined below :

‘the fat that in the eyes of millions of your countrymen you are a great patriot and great leader’ but ‘as a man subject to the law, who has, by his own admission broken the law’ you are subject to six years imprisonment

Code :

(a) C.R. Das
(b) Mahatma Gandhi
(c) Subhash Bose
(d) Jawaharlal Nehru

119. Consider the following statement :

‘We believe that it is the inalienable right of the Indian people as of any other people, to have freedom and to enjoy the fruits of their toil and have the necessities of life so that they may have full opportunities of growth’ Identify the correct context of this statement from below :

(a) Pledge of independence to be publicly taken all over India On 26 January 1930
(b) Preamble of the Constitution of India adopted in 1950
(c) Congress Working Committee Resolution adopted at the Special Calcutta Congress in 1920
(d) Part of Speech delivered by Subhash Bose at the launch of Azad Hind Fauj

120. Which one among the following led to the Greece economic crisis 2010 ?

(a) Excessive borrowing from IMF
(b) Sudden depreciation of Euro
(c) Unrestricted spending and cheap loans
(d) Outflow of foreign capital due to political unrest

81. (c) 82. (a) 83. (c) 84. (a) 85. (c) 86. (b) 87. (d) 88. (a) 89. (c) 90. (c)
91. (a) 92. (c) 93. (c) 94. (b) 95. (c) 96. (c) 97. (b) 98. (c) 99. (a) 100. (a)
101. (a) 102. (b) 103. (c) 104. (c) 105. (a) 106. (a) 107. (d) 108. (c) 109. (b) 110. (a)
111. (d) 112. (a) 113. (c) 114. (c) 115. (c) 116. (d) 117. (d) 118. (b) 119. (a) 120. (c)

Study Material for UPSC NDA/NA Examination

Books for UPSC NDA/NA Examination

Courtesy : UPSC

 

NDA/NA Exam Solved Papers - 2011- I - Paper I: General Ability Test (Part-B)

NDA/NA Exam Solved Papers - 2011- I - Paper I: General Ability Test (Part-B)

Paper I : General Ability Test

Part A : English Language

Comprehension

Directions (Qs. 41 to 50) : Each of the following items consists of a sentence followed by four words or group of words. Select the synonym of the word (occurring in the sentence in capital letters) as per the context.

41. The song had a SOPORIFIC effect on the child.

(a) soothing
(b) terrific
(c) supreme
(d) sleep-inducing

42. His ALLEGIANCE to the Party was suspect from the very beginning.

(a) servility
(b) obedience
(c) loyalty
(d) passivity

43. Sympathetic criticism has a SALUTARY effect.

(a) premature
(b) terrible
(c) disastrous
(d) beneficial

44. His bad behaviour EVOKED punishment.

(a) escaped
(b) called for
(c) produced
(d) summoned

45. Please do not give any more FLIMSY pretexts for not-having done the work.

(a) weak
(b) strong
(c) justified
(d) impulsive

46. He was the court appointed lawyer for fifty-six INDIGENT defendants.

(a) Indian men
(b) poor
(c) guilty
(d) untried

47. The boy said that pain had ABATED.

(a) reduced
(b) vanished
(c) increased
(d) stabilized

48. He became known as an IMPLACABLE foe of fundamentalism.

(a) relentless
(b) pleasing
(c) dangerous
(d) courageous

49. The queen was aware of the INSOLENT behaviour of the lords.

(a) violent
(b) polite
(c) insulting
(d) frivolous

50. Modern man is PRAGMATIC in his dealings.

(a) practical
(b) playful
(c) causal
(d) clever

Part B : General Knowledge

51. Which one among the following is not a reason for practicing tank irrigation in the Peninsular India?

(a) The undulating relief and hard rocks
(b) Little percolation of rain water due to impervious rock structure
(c) Most of the rivers of Peninsular India are perennial
(d) There are many streams which become torrential during rainy season

52. Identify from the following states of India through which the Tropic of Cancer passes and arrange them from east to west :

1. Gujarat
2. West Bengal
3. Uttar Pradesh
4. Jharkhand
5. Madhya Pradesh
6. Bihar
7. Chhattisgarh

Select the correct answer using the code given below :

(a) 2 - 5 - 7 - 4 - 1
(b) 2 - 4 - 7 - 5 - 1
(c) 3 - 2 - 6 - 7 - 5
(d) 3 - 7 - 4 - 6 - 2

53. Hot deserts like Sahara, Arabia etc. receive very negligible amount of rainfall. This is because they :

(a) do not receive moisture-bearing wind from the oceans.
(b) are the most rocky and barren areas of the Earth.
(c) are located on the tropical high’ pressure belt of the atmosphere.
(d) are not on the path of the monsoons.

54. The latitude is the angular distance of a point on the Earth’s surface, north of south, of the Equator as measured from the :

(a) centre of the Earth.
(b) Equator.
(c) Tropic of Cancer of the Capricorn.
(d) Poles.

55. When an Ordinary Bill is referred to a joint sitting of both the Houses of Indian Parliament, it has to be passed by a :

(a) simple majority of the total number of members of both the Houses present and voting.
(b) two-third majority of the total number of members of both the Houses.
(c) simple majority of the total number of members of both the Houses.
(d) two-third majority of the total number of members of both the Houses present and voting.

56. The Rowlatt Act was passed to :

(a) bring about agrarian reforms.
(b) curtail the nationalist and revolutionary activities.
(c) have a favourable balance of trade.
(d) put Second World War criminals on trial.

57. Which one among the following features of the Constitution of India is indicative of the fact that the real executive power is vested in the Council of Ministers headed by the Prime Minister ?

(a) Federalism
(b) Representative Legislature
(c) Universal Adult Franchise
(d) Parliamentary Democracy

58. In which among the following cases the joint session of both the Houses of Parliament can be summoned?

1. To amend the Constitution
2. When a bill has been pending with one House for more than six months after it was passed by the other
3. When both the Houses disagree on the amendments to be made in a bill
4. When a bill is passed by one House and is rejected by the other

Select the correct answer using the code given below :

(a) 1, 2 and 3
(b) 2, 3 and 4
(c) 2 and 3 only
(d) 1 and 4

59. The ‘Bombay Plan’ drafted by G.D. Birla and J.R.D. Tata emphasized :

(a) that the economy should be left to the dynamic investments by the private sector in heavy industries etc.
(b) the public sector investment in infrastructure and heavy industries.
(c) annual planning
(d) that the private sector should foot the bill for intensive and low return investments in the industrial sector.

60. Consider the following statements :

1. The Buddhists in Deccan excavated rock- cut Chaityas and the Vaishnavas, Shaivas and Jainas imitated these in later centuries.
2. The Vaishnavas, Shaivas and Jainas excavated temples at sites far distant from rock-cut Chaityas.

Which of the statements given above is/are correct ?

(a) 1 only
(b) 2 only
(c) Both 1 and 2
(d) Neither 1 nor 2

61. Consider the following statements :

1. The Champaran Satyagraha marked Gandhiji’s second appearance in India politics as a leader of the masses.
2. The Champaran Satyagraha was launched to address the problems faced by Indigo plantation workers.

Which of the statements given above is/are correct ?

(a) 1 only
(b) 2 only
(c) Both 1 and 2
(d) Neither 1 nor 2

62. Consider the following statements :

1. Charles Wood’s Despatch of 1854 laid exclusive emphasis on the development of higher education in India and neglected primary and secondary education
2. The Carlyle Circular issued by R.W. Carlyle sought to check the spread of revolutionary activities in educational institutions.

Which of the statements given above is/are correct?

(a) 1 only
(b) 2 only
(c) Both 1 and 2
(d) Neither 1 nor 2

63. Consider the following statements about Gandhiji’s thinking on environment :

1. His environmental thinking is rooted in his larger philosophical and moral thinking
2. He preferred sustainable environmental practices to nourish the soil and the natural world
3. He laid emphasis on the rigorous ethic” of non-injury in our treatment of animals

Which of the statements given above is/are correct ?

(a) 1 only
(b) 1 and 2 only
(c) 1, 2 and 3
(d) 2 and 3 only

64. Which one among the following is common to the treaty of Yandaboo (1826), the treaty of Salbai (1782) and the treaty of Gandamak (1879) ?

(a) With these treaties various Indian powers formed alliances to defeat the British
(b) These treaties enabled the British to control the South Asian powers
(c) These treaties expedited the spread of Indian culture abroad
(d) These treaties gave an essential boost to enhanced trade in South Asia

65. Which one among the following was the major demand of the Bardoli Satyagraha (1928) organized under the leadership of Sardar Vallabhbhai Patel?

(a) Land to the Tiller
(b) Increase in the rates of labour wage
(c) Rollback of newly enhanced revenue rate
(d) Supply of agricultural inputs to the farmers at subsidised rate

66. Which one among the following statements is not correct?

(a) Gandhara Schood of Art owed its origin to the Indo-Greek rulers but the real patrons of the school were the Kushans, especially Kanishka
(b) Rich carving, elaborate ornamentations and complex symbolism were not the main features of the Gandhara sculpture
(c) The Graeco-roman architectural impact modified the structure of Buddhist Stupas
(d) The artists of the Amravati School of Arts mainly used white marble

67. Which one among the following statements regarding the Eighth Five Year Plan in India is not correct?

(a) The plan was postponed by two years because of political upheavals at the centre
(b) It aimed at high growth of both agriculture and manufacturing sectors
(c) Its emphasis was on growth in export and import, improvement in trade and current account deficit
(d) It set before itself the two principal objectives of ‘growth with stability’ and ‘growth with justice’

68. Which among the following statements is/are true with regard to WTO membership?

1. All WTO members automatically receive the ‘most favoured nation’ status
2. Over 75% of WTO members are from developing countries. WTO membership allows them access to developed markets at the lower tariff

Select the correct answer using the code given below :

(a) 1 only
(b) 2 only
(c) Both 1 and 2
(d) Neither 1 nor 2

69. TRIPS (Trade Related aspects of Intellectual Property Rights) agreement is administered by :

(a) United Nations Conference on Trade and Development (UNCTAD)
(b) United Nations Organization (UNO)
(c) Word Trade Organization (WTO)
(d) World Bank (WB)

70. Consider the following statements :

1. The G20 was established in 2008 in the wake of the global financial crisis led by the USA
2. It brings together the major advanced and emerging economies to stabilize, the global financial market
3. India’s stand on Mutual Assessment Process (MAP) for measuring imbalances between surplus and deficit economies in the Seoul conference of G20 in the year 2010 was firmly endorsed by all the nations

Which of the statements given above is/are correct ?

(a) 1, 2 and 3
(b) 1 and 2 only
(c) 2 and 3 only
(d) 3 only

71. Which one among the following statements is not correct ?

(a) In progressive waves, the amplitude may be constant and neighbouring points are out of phase with each other
(b) In air or other gases, a progressive anti node occurs at a displacement node and a progressive node occurs at a displacement antinode
(c) Transverse wave can ‘Be’-polarized while longitudinal wave can not be polarized
(d) Longitudinal wave can be polarized while transverse wave can not be polarized

72. A body initially at rest is acted upon by a constant force. The rate of change of its kinetic energy varies :

(a) linearly with square root of time.
(b) linearly with time.
(c) linearly with square of time.
(d) inversely with time.

73. A jet plane flies through air with a velocity of 2 Mach. While the velocity of sound is 332 m/s the air speed of the plane is :

(a) 166 m/s
(b) 66kkm/s
(c) 332 m/s
(d) 664 m/s

74. Which one among the following statements is correct ?

(a) Convex mirrors are used by doctors to examine oral cavity
(b) Concave mirrors ‘are used as reflectors
(c) Convex mirrors are used as reflectors
(d) Convex mirrors should be used for shaving

75. Light travels in optical fiber irrespective of its shape because it is a device by which signals can be transferred from one location to another. It is based on the phenomenon of

(a) diffraction of light
(b) refraction of light
(c) polarization of light
(d) total internal reflection of light

76. A man is at rest in the middle of a horizontal plane of perfectly smooth surface of ice. He can move himself to the shore by making use of Newton’s :

(a) First law of motion
(b) Second law of motion
(c) Third law of motion
(d) First, second and third laws of motion

77. Which one among the following is the major cause of blurring and unsharp images of objects observed through very large telescope at the extreme limit of magnification?

(a) Air turbulence of earth’s atmosphere
(b) Poor optical polish achievable on large mirrors
(c) Poor tracking capacities of telescopes
(d) Varying density of air in the Earth’s atmosphere’

78. Bats can ascertain distances, directions, nature and size of the obstacles at night. This is possible by reflection of the emitted :

(a) ultrasonic waves from the bat
(b) ultrasonic waves from the distant objects
(c) supersonic waves from the bat
(d) supersonic waves from the distant objects

79. When a moving bus suddenly applies brakes, the passengers sitting in it fall in the forward direction. This can be explained by:

(a) the theory of relativity.
(b) Newton’s first law.
(c) Newton’s second law.
(d) Newton’s third ‘law.

80. The material used for electric fuse is an alloy of tin and lead. This alloy should have :

(a) high specific resistance and low melting point.
(b) low specific resistance and high melting point.
(c) low specific resistance and low melting point.
(d) high specific resistance and high melting point

41. (d) 42. (c) 43. (d) 44. (b) 45. (a) 46. (b) 47. (a) 48. (a) 49. (d) 50. (a)
51. (c) 52. (b) 53. (d) 54. (b) 55. (a) 56. (b) 57. (b) 58. (b) 59. (b) 60. (a)
61. (b) 62. (b) 63. (c) 64. (b) 65. (c) 66. (b) 67. (d) 68. (d) 69. (c) 70. (c)
71. (d) 72. (b) 73. (d) 74. (c) 75. (d) 76. (d) 77. (c) 78. (a) 79. (b) 80. (c)
.

Study Material for UPSC NDA/NA Examination

Books for UPSC NDA/NA Examination

Courtesy : UPSC

 

 

Pages

Subscribe to RSS - support's blog